Вы находитесь на странице: 1из 147

NBPME PART II RAPID REVIEW

H. Huynh NBPME Part II Rapid Review Congenital Hip Dislocation - Femoral head is usually posterior and superior to the acetabulum shortening of the limb with an increase in thigh folds - Limited hip abduction on the affected side - Appropriate handling and disposal of sharps - Sharps injuries are so commonly associated with occupational transmission of HIV and HBV sharp management is the best method to prevent infection - Surgical syndactylization of the fourth and fifth toes provides stability and a predictable position for a flail fifth digit - Vancomycin and Flagyl

Best method to prevent HIV infection and/or hepatitis B infection

Flail toe condition of the 5th

Abx therapy for pseudomembranous colitis Congenital vertical talus

- Presents as the talus fixed in a vertical position with hypoplasia of the talar neck and head - Navicular is dislocated and articulates with the dorsal aspect of the talar neck - Tibionavicular and dorsal talonavicular ligaments are contracted preventing reduction of the navicular. - The peroneus brevis tendon inserts on the styloid process of the 5th metatarsal and can cause fracture of the process with excessive traction applied to the bone by the tendon

Fx of the styloid process of the 5th metatarsal

H. Huynh NBPME Part II Rapid Review Tx of a navicular stress fracture (most important factor) - Uncomplicated navicular stress fractures will usually heal with strict nonweightbearing for 6 to 8 weeks - Surgical treatment is usually not recommended unless the navicular is unstable

Flat foot - Radiographic hallmark

- Transverse plane dominant - Increase T-C angle (AP) - Increase cuboid abduction angle (AP) - Decrease FF adduction angle (AP) - Decrease T-N congruency - Frontal plane dominant - Wide lesser tarsus (AP) - Dec 1st met declination angle (LAT) - Dec height of sustentaculum tali - Inc superimposition of lesser tarsus on LAT - Sagittal plane dominant - Inc talar declination angle (LAT) - Navicular cuneiform breach - Inc T-C angle (LAT) - Dec calcaneal inclination angle (LAT) - Transverse - Evans calcaneal osteotomy - C-C joint distraction arthrodesis - Kidner procedure - Frontal - Orthotics - Chambers bone graft under sinus tarsi - Baker-Hill osteotomy inferior to STJ posterior facet with bone graft - Selakovich (open wedge osteotomy of sustentaculum tali with graft - Silver/Dwyer calcaneal osteotomy

- Correction

H. Huynh NBPME Part II Rapid Review - STJ blocking - Sagittal - Lowman T-N fusion - Hoke N-C fusion - Miller N-C-M fusion - Cotton opening wedge cuneiform - Lapidus 1st met-C fusion - Young TA through keyhole in navicular - Cobb - TAL/ Gastroc resection Atopic dermatitis - The presence of asthma along with persistent, pruritic, vesiculating, erythematous eruption of both feet - Transverse plane deformity - Contributive conditions - A negative IM angle - Excision of the fibular sesamoid - Aggressive medial capsulorrhaphy - The extent to which a podiatric physician can practice is determined by the state legislature in a set of laws called the State Practice Act - An enzyme found in heart, brain, and skeletal muscle tissue - It is used to detect muscle disorders in serum blood testing - Most sensitive blood test for neuromuscular disorders. - Carry the sensory nerve fibers for position, vibration, and proprioception

Hallux varus

Scope of practice of podiatric medicine

Creatine Phosphokinase

Posterior columns of the spinal cord

H. Huynh NBPME Part II Rapid Review MRI impression of osteomyelitis - MRI T2-weighted images of OM have an increased signal intensity

MRI signal characteristics of body tissues (low signal = dark, high signal = white) - Low signal on T1-weighted images - High signal on T1-weighted images - Low signal on T2-weighted and STIR images - High signal on T2-weighted and STIR images Supination External Rotation (SER)

- Most fluid and muscle - Fat, some stages of hematoma, high protein-content fluid, slow-moving blood - Fat, some stages of hematoma - Fluid includes cysts, infection, acute fractures, chronic inflammation - The most common mechanism of a fractured ankle - At the time of injury, the foot is in a supinated position and an external rotatory force is applied. - Intrinsic pathway (PITT) factors VIII, IX, XI, XII - Monitor heparin therapy - Typical normal PTT range is 28-30 sec - Effective prophylaxis against DVT is typically considered to require a 1.5 fold to 2.5 fold elevation in the PTT - PTT is elevated when an inadequate level of Factor VIII is present - The most appropriate view to evaluate the posterior aspect of the ankle - The posterior process of the talus is clearly visualized on the lateral view of the foot and ankle

Partial throboplastin time (PTT)

LAT view radiograph

H. Huynh NBPME Part II Rapid Review Plaster of Paris vs. synthetic materials in the application of SLC - The primary advantage of plaster of Paris compared to synthetic materials is that it is more moldable - Appropriate cleansing agent for a surgeon who is allergic to iodine - Does not contain iodine - Sufficient for surgical preparation - Patients with AIDS have been found to be protected by the Americans with Disabilities Act. - It is illegal to discriminate against these patients by refusing to treat them or referring them to others based on their HIV status - Found in an individual with bronchial asthma triggered by an allergic reaction - Allergic or atopic diseases are some of the most common causes of eosinophilia - It is compensated for by subtalar joint supination - Rigid forefoot valgus that cannot be compensated for by forefoot inversion and first ray dorsiflexion will require supination of the subtalar joint - A biomechanical condition caused by overuse and is best treated mechanically - Mechanical support initially includes shoe modification, stretching, and taping - Mechanical support later includes orthotics to support the plantar fascia

Hexachlorophene

AIDS patients

Elevated eosinophils in the WBC differential

Rigid forefoot valgus

Heel spur or plantar fasciitis

H. Huynh NBPME Part II Rapid Review Hypotension during spinal anesthesia - The result of arterial and venous dilation - Sympathectomy may cause vasodilation - The forefoot-to-rearfoot relationship can be visualized from behind, allowing the STJ to be manipulated through its ROM to help determine the joints neutral position - The level of practice of the average, prudent provider in any given community - Digitalis poisoning can cause a variety of arrhythmias - Since calcaneal inversion is a component of subtalar joint supination, placing the heel in an inverted position also puts the subtalar joint in a supinated position, thus limiting STJ pronation - New bone formed beneath an elevated Periosteum surrounding necrotic bone in OM - Walled-off portion of sclerotic avascular bone in OM - Defect or sinus in cortical bone for the discharge of pus and debris in OM - Causes of wound dehiscence: - Infection - Hematoma - Injury to the wound - Incorrect suturing technique

Prone suspension casting technique

Standard of care

Digitalis (medication)

Balancing the heel of a positive cast in an inverted position

Involucrum

Sequestrum

Cloaca

Wound dehiscence

H. Huynh NBPME Part II Rapid Review Meningitis - The symptoms of acute meningitis include fever, headache, stiff neck - Showed on lateral WB radiograph of a flexible flatfoot - A drug used prophylactically against post-op nausea and vomiting - IV route - Used to treat enthesopathy - Reduces tightness and spasms, decreases inflammation, and assists in healing - A long leg cast with the ankle plantarflexed 20 degree ill immobilize the medial and lateral heads of the gastrocnemius muscle across the knee and also reduce the tension of the Achilles tendon distally - A long leg cast with the foot in an equinus position (a long leg cast immobilizes both knee and ankle joints, reducing strain on the gastrocnemiussoleus complex and Achilles tendon; casting the foot in an equinus position reduces the risk that the tendon will heal in an elongated, weakened position - Evolve from stem cells with the specialized ability to move toward and completely engulf bacteria and fungi - They are capable of producing a number of different granules containing potent chemicals, which results in the destruction of the engulfed organisms

Talar plantarflexion

Promethazine

Ultrasound therapy

Casting for patient who has undergone repair for an Achilles tendon rupture

Neutrophils

H. Huynh NBPME Part II Rapid Review - Primary function of neutrophils in the bodys defense is to phagocytize and destroy microorganisms Phases of wound healing - Inflammatory phase (substrate phase) the release of vasoactive amines and initial vasoconstriction - Fibroblastic phase (proliferative phase) epithelization, wound contraction, connective tissue repair period - Differentiation phase (remodeling phase) resorption phase - Number of deaths during a year divided by the number of live births that year. - The rate is expressed per 1,000 live births - Malignant bone tumor - Pulmonary embolism, myocardial infarction, dissecting aortic aneurysm - Not usually be localized to the substernal area. - Normally present as moderate, generalized chest pain - A potentially life-threatening characterized by extremely high fever, muscle rigidity, cardiac arrhythmias, and acidosis. - It may be precipitated by inhaled anesthetics, both depolarizing and nondepolarizing, neuromuscular blocking agents and stress

Mortality rate

Ewings sarcoma Causes of severe substernal chest pain radiating to the back Costal chondritis

Malignant hyperthermia

H. Huynh NBPME Part II Rapid Review Inversion stress test - Perform with the ankle in a neutral position is useful in the detection of injury to the calcaneofibular ligament - The calcaneofibular ligament is specifically stressed when the ankle is in a neutral position and inversion force is applied against the foot relative to the tibia. - Nail changes that is the characteristic of lichen planus - Frequently seen when nails are affected by lichen planus due to involvement of the nail matrix - When a radiograph is taken, current is measured in milliamperes (mA) - Increasing the current increases the number of electrons emitted, which in turn increases the intensity of the rays produced, thus resulting in a blacker radiograph - Kilovoltage control the contrast of a film - Increasing the kilovoltage produces a more penetrating x-ray but poor contrast - Increasing the kVp results in less exposure to the patient but a lower quality picture - Decreasing the kilovoltage will increase contrast - Should be measured with the rearfoot in neutral calcaneal stance position

Pterygium

Milliamperes (mA)

kVp

Tibial varum

H. Huynh NBPME Part II Rapid Review Lumbrical to the fourth toe - The tendon that is exposed during neuroma surgery performed on the third interspace through a dorsal approach - In the absence of any hypercoagulable state or significant risk factors, the medical compression provided by elastic stockings is usually sufficient to prevent blood clot formation postoperatively. - Used for the treatment of arterial insufficiency - Can be used to control premature ventricular contractions - Helps suppress ectopic ventricular rhythms by depressing automatically which decreases the slope of phase IV depolarization - Shortest duration of action local anesthetics - Commonly used for peripheral nerve blocks and has a usual duration of 1-3 hours - Cardinal sign: an increase in the thickness of the calcaneal fat pad - Enlargement of the hands and feet is an early feature - Wind: atelectasis, aspiration pneumonia, PE - Wound: surgical site infection, thrombophlebitis (IV site), pain - Water: UTI, dehydration, constipation - Walk: DVT - Wonder drugs: antimicrobials, and heparin

Elastic stockings

Pentoxifylline

Lidocaine

Acromegaly

The 5 Ws of post-op fever

10

H. Huynh NBPME Part II Rapid Review Fever - 0-6 hrs post-op - 24-48 hrs post-op - 72+ hrs post-op - Pain, anesthesia rxn, rebound cold from OR, endocrine causes - Atelectasis, aspiration pneumonia, dehydration, constipation - Infection (3-7 days), DVT, UTI, drug allergy, thrombophlebitis - A complete sequential reduction will reduce the deformity, but the position must be maintained with temporary fixation to prevent redislocation by stabilizing the digit to the base of the proximal phalanx and across the MPJ with the K-wire - Symptoms often include pain, swelling, a flattening of the arch, and an inward rolling of the ankle - These symptoms do not improve with conservative treatment such as shoe modification - Also known as peroneal muscle atrophy - As disease progresses, symmetrical muscular atrophy and weakness are apparent in peroneal muscles and toe extensors. - Increases the Spasticity of peroneus longus muscle, resulting in the supination of the entire foot around the STJ, producing pes cavus appearance of the foot - Set public health policies and standards - Collect, analyze, and disseminate health information

Sx for dislocated hammertoe

PT tendon rupture

Charcot-Marie-Tooth Disease

State departments of health

11

H. Huynh NBPME Part II Rapid Review - Health departments pursue a legislative policy agenda promoting health. They also design and implement a health care delivery system Sesamoid bones - A type of accessory bone that is embedded within a tendon or joint capsule - Sesamoid bone within the peroneus brevis tendon insertion at the 5th metatarsal base - Posteromedial aspect tuberosity of navicular, within insertional fibers of tibialis posterior - Agitation - Nocturnal disorientation (sundowning) - Urinary incontinence - Dehydration - Increase respiration rate - Respiratory alkalosis - Medial capsulotomy - Exostectomy - Section of the deep transverse intermetatarsal ligament - Release of the conjoined adductor tendon - Removal of the fibular sesamoid - An IV sedative-hypnotic that is used as an induction agent - Appears as dryness and scaling in a moccasin distribution

Os peroneum

Accessory navicular type I (os tibiale externum)

Potential signs of early sepsis in the elderly

Steps in McBride bunionectomy

Thiopental

Tinea pedis

12

H. Huynh NBPME Part II Rapid Review Cigarette smoking - The use of tobacco is the most prevalent and preventable risk factor involved in LE ischemia and disease - Single most important preventable risk factor for peripheral vascular disease - Causes: vertical talus, gastrocneminus equinus, and posterior tibial tendon rupture - The film is placed vertically in the orthoposer with the medial aspect of the foot and ankle against the plate. - The patient stands with the medial side of the foot placed against the film and the tubehead angled 90 degrees from vertical - Extrinsic pathway (PET) factors III, VII - Used in the management of patients on warfarin - Measure blood clothing ability and is increased with the use of warfarin - The most appropriate antibiotic prophylaxis when implant surgery is performed - Most effective against Staph. Aureus, the most common cause of postoperative infection - Useful in the treatment of a patient with ankle fusion secondary to trauma - Acts as a dynamic lever to decrease the need for required ankle motion in the sagittal plane during the gait cycle, allowing the patient to ambulate with a

Pes planus

WB Lateral View

Prothrombin time (PT)

First-generation cephalosporins

Rocker-bottom shoe

13

H. Huynh NBPME Part II Rapid Review more normal gait Stress fracture of the metatarsal - Can occur after excessive cyclic loading of the bone, such as after prolonged, strenuous weightbearing activity - Swelling and erythema are seen in the initial phase

Charcot arthropathy

Isolated rupture of the calcaneofibular ligament

- Ankle stress radiographs will reveal a negative anterior drawer sign and positive talar tilt - Allow the talus to invert relative to the tibia due to lateral instability but will not allow anterior displacement of the talus to the tibia if the anterior talofibular ligament is intact - Should be measured preoperatively and postoperatively. - The need for additional measurements is determined by the duration and magnitude of surgery and the stability of the diabetes - Signs of hypoglycemia include tachycardia, hypertension, and diaphoresis - Increased talar declination angle - Increased talocalcaneal angle - Anterior break in the cyma line - Radiograph shows increased metatarsus adductus angle

Blood glucose levels

Radiographic features of closed-kineticchain STJ pronation

Metatarsus adductus

14

H. Huynh NBPME Part II Rapid Review Child abuse report - Immune the physician from prosecution arising from reporting suspected child abuse in good faith. - X-ray findings include dorsal talar head and neck beaking - Pain is a common symptom - Limitations of STJ motion and valgus deformity vary in severity - The Harris-Beath (calcaneal axial) view can allow visualization of the middle facet of the talocalcaneal joint. The joint space is obliterated in the case of osseous coalition. If there is fibrocartilaginous coalition, the joint space appears narrow and subchondral sclerosis may be present. - Occurs in the peripheral arteries of the lower limbs as calcification of the tunica media. - The radiographic appearance has been called rail tracking and described as a pipe stem pattern - Radiographic features include dual, curvilinear soft tissue calcifications that are parallel to one another and somewhat serpiginous in appearance. - High frequency sound waves are absorbed primarily by connective tissue, such as tendons and ligaments, heating the tissue, increasing blood flow, and reducing chronic inflammation - Sonic energy increases the temperature in deep tissues

Tarsal coalition

Monckebergs arteriosclerosis

Therapeutic ultrasound

15

H. Huynh NBPME Part II Rapid Review Osteogenesis imperfecta - A genetic disease that results in weakness of the bones secondary to a malfunction of the bodys production of collagen. - Pedal manifestations include decreased metatarsal girth - The result of antigen binding to IgE, which is attached to circulating tissue mast cells and basophils. - Common radiographic findings include subchondral sclerosis, asymmetric joint space narrowing, and marginal osteophyte formation - Assessment for unresponsiveness as the initial action to be performed. - Degenerative disease of the subtalar joint (with a calcaneonavicular coalition requires arthrodesis) - Clinical features include warm moist skin, lid lag, and fine tremors - Assisted by the plantar intrinsic musculature, which includes the quadratus plantae - Running in a shoe with a sole that is too rigid across the MPJs increases a persons risk of developing - A shoe that does not flex at the MPJs has the effect of increasing the lever arm of ground reactive force at the ankle. The longer the lever arm of ground reactive force, the greater the

Immediate hypersensitivity

Osteoarthritis

Basic life support (BLS)

CONTRAINDICATION to the resection of a calcaneonavicular coalition

Hyperthyroidism

Stabilization of the toes in midstance

Anterior tibial shin splints

16

H. Huynh NBPME Part II Rapid Review tension on the Achilles tendon in plantarflexing the ankle. This results in overuse of the tibialis anterior muscle as it attempts to counteract the resistance of the Achilles tendon, causing inflammation where the muscle of the tibialis anterior attaches to the anterior aspect of the tibia Hinge axis concept - Useful when planning a metatarsal basilar osteotomy - Maintaining the hinge axis for a metatarsal basilar osteotomy is critical to provide another point of stabilization - IV barbiturates can produce hypnosis, unconsciousness with increased doses, and interference with the sensitivity of the medullary respiratory center to carbon dioxide. - Cardiovascular effects of barbiturates include a decrease in blood pressure and a compensatory increase in heart rate - A fixed periodic HMO payment calculated over the expected cost of providing services to patients over a period of time. - Superficial

Barbiturates

Capitation

When an excision is performed through a plantar approach, the two proper digital nerves that are divisions of the common digital nerve are found ________ to the deep transverse metatarsal ligament Common iliac artery occlusion

- Symptoms include weak femoral pulses, sexual dysfunction, buttock claudication

17

H. Huynh NBPME Part II Rapid Review Achilles tendon rupture - A primary anastomosis with an augmentation graft will have a high rate of success in young healthy athletes - The patient is at risk for delayed union or nonunion of the fracture due to instability and limited blood supply in the region of the fracture. Nonweightbearing is a necessary part of the conservative post-injury treatment to allow for complete healing in a timely manner. - Clinical signs include: alopecia, hypertension, and temporary sterility - FDA-approved for use to relieve the bone pain associated with osteoporosis - A form of resistance training where the muscle contracts but does not change length while exerting a force, allows muscles to be exercised while the limb is immobilized such as patient is in short leg cast - A flexor to extensor tendon transfer for stabilization requires healing with the proximal phalanx plantarflexed at the metatarsophalangeal joint to ease tension on the tenodesis - Inhaled beta-adrenergic agonists provide immediate relief of asthma symptoms by causing direct dilation of the constricted air passages

Nonweightbearing cast

Anabolic steroid use in young athletes

Isometric exercise

Flexor to extensor tendon transfer

Initial tx of an acute asthmatic attack

18

H. Huynh NBPME Part II Rapid Review Opioid agonists (hydrocodone) abuse - S/S includes miosis, drowsiness, and pruritus of the nose - Cause constipation - Important in the evaluation of a flatfoot deformity. - When the talocalcaneal angle is markedly increased, heel valgus is said to be present, which indicates a more significant deformity. - Increase in a flatfoot deformity Under civil law, the performance of a procedure without consent is an offense referred to as battery - Gait pattern most commonly seen in cases of lower motor neuron pathology - Lower motor neuron lesions cause weakening of muscles and paralysis which leads to a steppage gait - Turn the patient on his side because the greatest immediate risk to the patient is aspiration of vomit which is minimized when the patient is turned on the side. - Typical radiographic finding is osseous protuberance with the cortex continuous with the parent bone - A key radiographic feature that defines an osteochondroma is the continuity of the cortex of the lesion with the cortex of the involved bone

Talocalcaneal angle

Battery

Steppage gait

Emergency seizure management in office

Osteochondroma

19

H. Huynh NBPME Part II Rapid Review Standard anode-film distance in podiatric radiology - The standard distance between the film and anode when a podiatric foot xray is performed is 24-30 inches - Common benign tumors but some pigmented basal cell carcinomas and even some melanomas can present with similar appearance - Excision with pathological examination is often indicated and is the best treatment - May be waived by the patient only - In many states, the doctor can invoke a legal privilege on the patients behalf when asked to disclose or divulge information about the patient. This privilege belongs to the patient, not the doctor, so only the patient may waive it, usually by written consent - After a general anesthetic is administered, a minimal level of monitoring must be provided in the postanesthesia care unit. - Vital signs including blood pressure, heart rate, breathing rate, airway patency, and level of consciousness should be monitored every 15 minutes - All three tests elevated suggest myopathy or a neuromuscular disease such as muscular dystrophy - The mechanism of injury produces an intra-articular fracture of the calcaneus, creating a posterior facet fragment that is impacted or

Dermatofibromas

Doctor-patient privilege

Vital signs following general inhalation anesthesia

Creatine Phosphokinase, aldolase, and alanine transaminase

Joint depression calcaneal fracture

20

H. Huynh NBPME Part II Rapid Review depressed into the calcaneal body Lesions will have the lowest signal intensity in a T1-weighted MRI of the foot - Tissues consisting primarily of water have the lowest signals on T-1 weighted images - e.g. ganglion cyst, unicameral bone cyst, adamantinoma - In AP view - Increase in the talo-first metatarsal angle from the normal range of 0 to -20 is due to the adductus of the forefoot - Decrease in the talocalcaneal angle (Kites angle) from the normal range of 20 to 50 reflects the inversion (varus) of the heel - Typical presentations include bone pain, renal failure, Normocytic anemia - Lytic lesions are frequently seen on bone scans as cold areas - Hypercalcemia is frequently seen due to increased bone destruction and release of calcium into the circulation

Talipes Equinovarus

Multiple myeloma

In serial casting for Talipes Equinovarus, - Adduction the components of the deformity are - Inversion (Varus) corrected in the order - Internal torsion at the tibia - Equinus Diazepam - In the anesthetic setting, diazepam is used as preoperative medication and adjuvant drug partly because of its anxiolytic properties - Diazepam is used in the anesthetic setting to reduce anxiety

21

H. Huynh NBPME Part II Rapid Review Bisphosphonates - A class of compounds that slow bone resorption and are used to treat osteoporosis - talar tilt test - Most useful for evaluating injury to the lateral ankle ligaments

Inversion ankle stress view

Very end of the midstance phase of gait, - The phase of gait during which the just before heel lift bodys center of mass is directly over the metatarsophalangeal joints and the sesamoids are most compressed - The metatarsophalangeal joints and sesamoids are regions that receive maximal ground reactive force at the very end of the midstance phase of gait, just before heel lift. Lipoma on MRI - A fat containing lesion like that has a short T1 signal and a long T2 signal - The most common mechanism of metatarsophalangeal dislocation in football players - Extreme dorsiflexion of the MPJ causes the metatarsal head to rupture the plantar joint capsule with dorsal and proximal displacement of the digit. The injury is common among football players due to the combination of artificial playing surfaces which require the use of more flexible shoegear and the frequency with which the player is positioned with the toes fixed on the playing surface and the heel raised.

Hyperdorsiflexion

22

H. Huynh NBPME Part II Rapid Review Hepatitis B infection - The main concern during surgery following an inadvertent scrape or needle stick - Patients with poorly controlled hypertension may be at an increased risk for intraoperative or postoperative myocardial infarct or stroke. - A traditional recommendation has been to delay surgery if the diastolic blood pressure is greater than 110 mmHg - The drug of choice for treatment of erythrasma which fluoresces coral pink under a Woods light. - Obesity, varicose veins, and smoking

Poorly controlled hypertension

Erythromycin

Risk factors for the development of deep vein thrombophlebitis Closed-chain kinetics STJ pronation

- Talus adducts and plantarflexes, and the leg internally rotates - The appropriate treatment when a patient with a cast complains of severe pain, bluish discoloration of the digits, numbness, and throbbing 12 hours postop. - The application of a bivalve cast will provide immediate relief of the patients symptoms and restore blood flow - The establishment of a doctor-patient relationship requires an individual to voluntarily seek a doctors care and expect that the communication be held in confidence. - The relationship can be defined in writing but can also be oral or implied.

Bivalve cast

Doctor-patient relationship

23

H. Huynh NBPME Part II Rapid Review Hypertrophic nonunion - Results from excessive motion, so eliminating movement will allow complete bone healing - Benefit most from immobilization alone - Patients with RA frequently have a subluxation of C1 and C2 - Clinical features of RA include tendon and ligament erosion which in the cervical spine leads to instability between the C1 and C2 vertebrae - The only muscle belly present in the first interspace - Oral treatment of onychomycosis - Fungicidal rather than fungistatic at the minimum therapeutic dose - Displacement after a Lisfrancs fracture most commonly occurs in a dorsal and lateral direction. The dorsal ligaments are intrinsically weak and more likely to rupture than the plantar ligaments. Lateral deviation occurs due to rupture of Lisfrancs ligament - Short leg cast with the ankle neutral will provide zero tension and ideal positioning for healing of a repaired anterior tibial tendon - OSHA regulations state that it is mandatory to post regulations in a location all employees routinely visit - A nonprogressive lesion in the brain that alters motor control, leading to

Rheumatoid arthritis

First dorsal interosseous muscle

Terbinafine

Lisfrancs fracture

Casting technique after the primary repair of a ruptured anterior tibial tendon

OSHA regulations

Cerebral palsy

24

H. Huynh NBPME Part II Rapid Review disorders of movement and posture - In order to classify a neuromuscular disorder as _________, there must be a nonprogressive lesion Inflammatory phase of wound healing lasts approximately ____________ - 3-5 days - Blood fills the wound, epithelial cells mobilize, and venules are more permeable - Appears at approximately 3 years of age - The last center of ossification to appear in the foot - Flexion-extension creases in the skin form tension lines perpendicular to the direction of muscle pull over joints - Physical therapy of the affected extremity - Inactivity can exacerbate the disease and perpetuate the pain cycle - Has strong, heavily constructed opposing jaws, each of which scooped out like the tip of a curette - A relatively safe, noninvasive method of pain management - Used for pain relief - Not recommended for a patient with sepsis due to the increased risk for meningitis - First lumbrical - First and second dorsal interossei

Ossification center of the navicular

Skin tension lines

Initial tx of choice for RDS

Rongeur forceps

Transcutaneous electrical nerve stimulation (TENS)

Spinal anesthetic

Muscles that insert at the base of the proximal phalanx of the second digit

25

H. Huynh NBPME Part II Rapid Review Clostridium perfringens - Most commonly associated with soft tissue emphysema or gas on a radiograph - Half of all osteoid osteomas are seen in the tibia and femur - Most suggestive of child abuse - Erythema nodosum and subcutaneous nodule formation are common in the latter stages of sarcoidosis - The presence of shortness of breath and nonproductive cough are also manifestation of sarcoidosis - May cause ankle pain - Type I Same (just physis) - Type II Above (spare epiphysis) - Type III Below (spare metaphysis) - Type IV Through (all 3 get hit) - Type V Really bad - Tibialis posterior muscle acts as a supinator of the STJ, counterbalancing the effects of the peroneus brevis muscle, which acts as a pronator - Posterior tibial tendonitis will result in weakness of the muscle, causing a more pronated appearing foot, as well as discomforting at the tendons insertion on the navicular tuberosity - Examination technique supination of the STJ against resistance (since the tibialis posterior muscle is a supinator of the subtalar joint, having the muscle contract against resistance should demonstrate both pain and weakness

Osteoid osteoma

Fractures in different stages of healing Sarcoidosis

Salter-Harris Classification of growth plate

Tibialis posterior tendonitis

26

H. Huynh NBPME Part II Rapid Review associated with tendon dysfunction. Rheumatic heart disease - Characterized by mitral valve damage - A common complication of rheumatic fever is thickening and stenosis of the mitral valve - The least time of radiation exposure while minimizing the current (mA) takes precedence over the kVp when trying to minimize radiation dose to the patient - The presence of asymptomatic central ulceration, which may be dry and crusted is highly suggestive of basal cell carcinoma - Frequently associated with sun exposure - Acidosis from local infection retards the diffusion of local anesthetics because of increased ionization - A T cell mediated phenomenon wherein the T cells are sensitized with exposure to a specific antigen, then activated by subsequent re-exposure to the antigen - Delayed cutaneous hypersensitivity - Scintigraphy records on film the distribution of radioactivity in tissue following the use of radioactive tracer substances - When a stress fracture is strongly suspected and the initial radiographs are normal, the next study to order would be a triple phase nuclear medicine bone scan

Lowest radiation dose to the patient

Basal cell carcinoma

Infection and local anesthetic

Allergic contact dermatitis

Bone scintigraphy

27

H. Huynh NBPME Part II Rapid Review Osteochondroma - An overgrowth of cartilage and bone - The most common benign bone tumor accounting for 20-50% of benign bone tumors and 10-15% of all bone tumors - Open reduction and internal fixation with an intramedullary screw - In a competitive athlete, this type of fracture should be treated aggressively due to the increased incidence of delayed union or nonunion associated with this injury - Hands below the waist - Defined as the number of electromagnetic waves that pass a given point per unit of time - 65 years and older - Health costs for those age 65 and older are more than three times higher than for younger age groups. - TE is always shorter than TR - TE is < 30 msec - TR is < 600 msec - TE is > 60 msec - TR is > 2000 msec - Chronic hematuria, aseptic necrosis of the hips, chronic Osteomyelitis caused by Salmonella - Sickle cell disease complications are due to vascular effects caused by the altered red blood cell shape and function

Tx of a Nondisplaced transverse fracture at the proximal metaphysealdiaphyseal junction of the 5th metatarsal in a competitive basketball player

Violation of sterile technique Wave frequency

In the United States, the largest percentage of health dollar is utilized in this age category

TE and TR scan parameters of T1 image on an MRI

TE and TR scan parameters of T2 image on an MRI Complications caused by sickle cell disease

28

H. Huynh NBPME Part II Rapid Review Normal first ray function and anatomy - The first ray axis runs through all three planes of motion from a posteromedial and dorsal direction to an anterolateral and plantar direction. - When the first ray dorsiflexes, it also inverts and adducts - PL stabilizes the first ray in midstance - Is regulated by laws that vary from state to state but generally define competent adults eligible to make health care decisions - Caused by blood pooling in the legs secondary to venous distention and a failure of the venous valves to maintain and upward flow of venous blood against gravity - Compression dressings are an effective way to compensate for this lack of proper venous function - An end product produced when hemoglobin glycosylates due to hyperglycemia - Assessment of hemoglobin A1c provides useful information related to a patients regulation of blood glucose during the previous 3 months - Most common location is 2 to 6 cm proximal to its insertion - Approximately 2 to 6 cm proximal to its insertion, the Achilles tendon has an area of decreased blood flow, making this region more vulnerable to strain and rupture

Informed consent

Venous stasis ulcers

Hemoglobin A1c

Achilles tendon rupture

29

H. Huynh NBPME Part II Rapid Review Stratum granulosum - One of the five layers of the normal plantar skin epidermis. - Lies just above the mucosal layer of the epidermis - Imaging modality that results in no ionizing radiation exposure to the patient - MRI uses nonionizing radio frequency pulses that are absorbed and then emitted by tissue lying inside a magnetic field - Risk factors such as smoking, immobilization, and oral contraceptive use. - Description of the leg as hot, red, and indurated is most consistent with DVT - Homans sign is a diagnostic sign, although unspecific for DVT - Hemi-implant acts as a spacer following bone removal - Removal of the proximal phalangeal base of the hallux may result in shortening and postoperative contracture of the hallux. Maintaining this space with a hemi-implant may reduce this complication - Symptoms: stridor, laryngospasm, wheezing, angioedema, hives, possible hypotension, and circulatory collapse. - The lack of oxygen due to bronchospasm is the primary cause of death

MRI

Deep vein thrombosis (DVT)

Hemi-implant use with Keller procedure

Anaphylaxis

30

H. Huynh NBPME Part II Rapid Review Most common adverse reaction to IV sedation with morphine - Respiratory depression - Morphine causes depression of ventilation - Runner does not have a double support phase of gait - Stance phase is shortened as running speed increases, which decreases the time in stance and results in the elimination of the brief time when both feet are in stance at the same time - Adhesion - Fibrous scar tissue often causes adhesions between a tendon and its sheath and the overlying skin - The government agency responsible for determining the fees and services that are eligible for payment for Medicare and Medicaid recipients - Federal law to provide that direction defines this agency - A high intratendinous signal intensity on a heavily water-weighted image is suggestive of an acute rupture or injury - On MRI, a rupture or an injury appears as an area of increased signal in the tendon - Most often associated with pes planus and peroneal spasm, with available motion seen usually in the pronatory direction - Findings are associated with a decreased calcaneal inclination angle

Running gait cycle

If a skin incision is made directly over a tendon, there is an increased risk of ________

The Centers for Medicare & Medicaid Services

MRI of the Achilles tendon

STJ coalition

31

H. Huynh NBPME Part II Rapid Review Pathogen most likely associated with bowel incontinence and an infected heel decubitus ulcer - E.coli - E.coli is transmitted from fecal material and isolated from stool cultures. In a patient with bowel incontinence, the ulcer is most likely infect from the patients fecal material - Four elements: - Duty - Breach - Causation - Damages - Tort is an act of wrongdoing, which is civil in nature rather than criminal - It is a wrong independent of contract or criminality - Health care providers have a duty to do what is right and reasonable - The elements of the tort of malpractice address situations in which there is a breach of duty that causes or results in harm - Mechanical debridement - When a wet-to-dry dressing is removed, superficial necrotic tissue is also removed from the wound - Pronation - The talus adducts and plantarflexes in closed chain pronation, and actually decreases the amount of articulation of the talus and navicular - Eliminate joint pain - The implant function as a spacer between the proximal phalanx and the first metatarsal after the painful joint

Tort of medical malpractice

Wet-to-dry dressing

In AP view, a talonavicular articulation of less than 70 percent is a sign of ________

The primary goal of implant surgery of the first metatarsophalangeal joint

32

H. Huynh NBPME Part II Rapid Review has been resected Sickle cell anemia - Symptoms include jaundice, chronic anemia, episodes of abdominal, back, and limb pain. - Sickle cell anemia manifests itself in infancy. - Management is largely symptomatic and supportive - Partially threaded screw - Used to create interfragmental compression by the lag principle. This technique is commonly used to provide the best compression and alignment for an Austin bunionectomy - Hepatic necrosis - In genetically susceptible patients, an oxidative trifluoroacetyl metabolite of halothane may evoke the production of neoantigens directed against hepatocytes - Eating disorders, amenorrhea, and osteoporosis - Affects women and girls in many sports - Muscle biopsy - Muscle strength evaluation - Genetic screening for a sex-linked recessive disorder - Diagnostic test for autonomic nervous system disorders

Cancellous screw

Complication of halothane anesthesia

Female athlete triad

Diagnostic tests for Duchenne muscular dystrophy

Acetylcholine receptor (AChR) antibody test

33

H. Huynh NBPME Part II Rapid Review Shoe wear pattern of a compensated structurally long limb - Compensation for a structurally long limb produces excessive pronation of the foot on the involved side the wear pattern of the shoe on the involved foot would, therefore, be greatest on the medial side - Heel counter tipping medially in a valgus position - The number of new cases of a specific disease occurring during a given period in a specific population - Hemarthrosis from hemophilia occurs in up to two-thirds of patients and is heralded by pain, warmth, and stiffness. - The knee, elbow, and ankle are the joints most frequently affected. - Performed by squeezing the calf muscle of a patient lying in the prone position, and observing for plantarflexion of the foot. - Absence of foot plantarflexion is indicative of disruption of the Achilles tendon - Ventricular fibrillation (V-fib)

Incidence

Inflammatory arthritis associated with hemophilia

Thompson test

Life-threatening dysrhythmias that occurs in the early phases of a myocardial infarction Nutrient artery of the first metatarsal Apophysis of the calcaneus

- Enters the midshaft laterally - Can be develop normally from multiple ossification centers with the adjoining metaphysis appearing sclerotic, irregular, or even serrated.

34

H. Huynh NBPME Part II Rapid Review Somatization disorder - A psychiatric diagnosis applied to patients who chronically and persistently complain of varied physical symptoms that have no identifiable physical origin - A neurotic illness characterized by: - Dramatic, emotional presentation of complaints - Multiple, vague complaints referable to any part of the body - Dissatisfaction with care from one physician to the next - Squamous cell carcinoma - Many types of chronic skin irritations and long-standing ulcers, such as venous stasis ulcers, can undergo malignant transformations, the vast majority of which result in well-differentiated squamous cell carcinoma - A large phagocytic cell that transforms into a macrophage once it enters tissues - Lymphocytes - Erythroblasts - Myeloblasts - Spermatogonia/ oocytes - Epithelial cells - Endothelial cells - Bone - Nerve - Brain - Muscle

Patient with long standing varicose ulcer over the medial malleolus is at higher risk for the development of _________

Monocyte

Cell sensitivity to radiation (most to least)

35

H. Huynh NBPME Part II Rapid Review Radiologic fixation process - Flaws in the fixation process are the most common cause of yellow discoloration of radiographic film - Cerebral palsy - Myelomeningocele - Charcot-Marie-Tooth disease - In muscular dystrophy, contractures of the gastrocnemius muscles appear early and result in tightening of heel cords causes the patient to have an equinus deformity and walk on the toes - Bence Jones proteins in the urine are diagnostic of multiple myeloma (a malignancy of plasma cells) - These proteins are produced by plasma cells and are considered the first tumor marker - Periosteal elevator - Immobilization with a short leg cast - Latent ankle diastasis requires to reduction and is treated by cast immobilization. Distal syndesmotic ligamentous injuries without fractures or gross widening of the ankle mortise are treated nonoperatively with a short leg cast, followed by physical therapy - Determine the presence of a talocalcaneal coalition that involves middle and posterior facets - Increased osteoblast and osteoclast activity

Neuromuscular causes of cavus foot

Foot deformity in muscular dystrophy

Bence Jones proteins

Sayre Tx for diastasis of ankle joint

Harris-Beath view

Pagets disease

36

H. Huynh NBPME Part II Rapid Review A 6 year old presents with a painful ankle injury of 3 days duration. There is swelling and pain on direct palpation of the lateral malleolus. X-rays are unremarkable. Tx of choice is _______ Dermatologic lesion associated with Lyme disease A runner who twisted her ankle 6 months ago still has ankle pain and an occasional sensation of the ankle giving away and locking. She has no apparent swelling or pain on range of motion. The most likely diagnosis is ____________ Pyoderma gangrenosum - Since there is no visible fracture on xray, an epiphyseal fracture of lateral malleolus is likely. A short leg cast for 3 weeks would be the treatment of choice - Erythema chronicum migrans

- Osteochondral talar dome fracture

- An autoimmune disease and can be caused by many systemic diseases, although the exact etiology is unknown - It is associated with inflammatory bowel diseases such as ulcerative colitis and Crohns disease - Anterior talofibular - Most inversion ankle sprain injuries occur when the foot is plantarflexed and inverted at the ankle joint, which results in instability. The anterior talofibular ligament is the first ligament of the lateral ankle area to be maximally stretched and torn with this type of injury - Initial joint space widening followed by a long-term increase in subchondral bone density

Ligament that is most commonly damaged in an inversion sprain of the ankle

Radiographic findings indicative of Freibergs infraction are ___________

37

H. Huynh NBPME Part II Rapid Review Freibergs infraction - True osteonecrosis of the second metatarsal head, with the inflammation causing the initial joint space widening. The resultant osteoarthritis causes the subchondral sclerosis commonly seen - S/S: ataxia, impaired vision, bladder dysfunction - Patients with multiple sclerosis do not usually have signs and symptoms affecting the lower motor neurons. - Lower extremity reflexes are generally hyperactive - A progressive deterioration of a joint characterized by peripheral neuropathy. - The goal of treatment is to maintain a stable foot in order to decrease the tendency of the foot to subluxate or dislocate without support - A custom molded shoe will help maintain the foot in a stable position and help protect the extremity from repetitive microtrauma and skin break down. - Osteoid osteoma

Multiple sclerosis

Charcot neuroarthropathy

Tumor that is associated with night pain that is relieved by salicylates An inhibitor of prostaglandins Cavernous hemangioma

- Aspirin - Pain and rapid increase in size of the lesion without trauma - X-rays show a calcification of portions of the lesion - An aggressive lesion is best treated with surgical curettage and packing

38

H. Huynh NBPME Part II Rapid Review Tzanck smear - Diagnostic test for Herpes Simplex - Herpes simplex is diagnosed with a Tzanck smear in which fluid from an intact vesicle is smeared on a glass slide, dried, and stained with Giemsas stain - Increases the incidence of osteosarcoma - The most serious complication of Pagets disease is sarcomatous degeneration. Osteosarcoma are the most common but fibrosarcomas and chondrosarcomas can also be seen - CT scan

Pagets disease

The best modality to confirm an osseous subtalar joint coalition Pulmonary emboli

- Usually released from deep venous structures of the lower extremity. They lodge in pulmonary vasculature, which causes a decrease in oxygen levels and an increase in pulmonary blood pressure. The result is air hunger, tachypnea, and dyspnea - Associates with drug-induced systemic lupus erythematosus - Patients who take procainamide for prolonged period often develop anticardiolipin antibodies as well as antiDNA and anti-histone antibodies. - Drug-induced systemic lupus erythematosus is rare and usually subsides once the medication is discontinued

Procainamide

39

H. Huynh NBPME Part II Rapid Review Melanocytes - Contained in the stratum germinativum, also known as the basal layer of the epidermis - Shin splints are a common injury in runners and are most symptomatic when a person is running on hills. - Overpronation is often the etiology - Chronic inflammatory disease characterized by proliferative synovitis with subsequent bone erosion and destruction of the articular cartilage - RA is characterized by synovial inflammation - Joint destruction in RA targets articular cartilage, ligaments, tendons, and bone - Conservative therapy that consists of rest, anti-inflammatory medication, and shoe modification - A disorder caused by nematode larvae, is serpiginous in appearance - Responds to treatment with thiabendazole - Limited to the posterior aspect of the heel and resolves by the time the apophysis fuses with the body of the calcaneus - A self-limiting disease - An effective and commonly used endto-end technique for the repair of tendon ruptures - Technique is used for tenorrhaphy

Anterior shin splints

Rheumatoid arthritis

First step in the treatment of a painful pes planus deformity

Cutaneous larva migrans

Severs disease

Bunnell technique

40

H. Huynh NBPME Part II Rapid Review Malignant melanoma - Produce the type of pigmented skin changes that could mimic Kaposis sarcoma - Bone lesion that is characterized by dense bone surrounding a lytic central nidus - A small benign osteoblastic tumor. It initially appears as a sclerotic bone island with a central lucent defect - Amoxicillin 3 gm orally 1 hour before the procedure and 1.5 gm orally 6 hours after - Most appropriate for rigid foot with fixed deformity - The goal of the semi-weight-bearing technique is to capture an accurate impression of the size and shape of the foot. - The cast produced via this technique is most appropriate for an accommodative orthosis - 3:1 - Generally, excisional biopsies for skin lesions are performed in the shape of an ellipse with pointed ends. The long axis of the ellipse should be roughly three times its central and greatest width to prevent buckling of the skin - An inverter of the foot - Support the arch and counteract the effect of peroneus brevis, an everter of the foot - One of the main characteristics of

Osteoid osteoma

Prophylaxis against Staphylococcus aureus and Staphylococcus epidermidis with NKDA Semi-weight-bearing casting technique

Standard length to width ratio for excisional biopsy

Posterior tibial muscle

41

H. Huynh NBPME Part II Rapid Review posterior tibial dysfunction is loss of the arch Pronation of the foot - In pronation, the talus is positioned more distally and plantarly, which increases the talar declination angle. The calcaneus is lowered and everted, which decreases the calcaneal inclination angle. - Manipulation of the kVp will affect radiographic subject contrast. - A lower kVp will increase the contrast; a higher kVp will decrease the contrast - Type I - A Salter-Harris type I fracture is simply a fracture of the growth plate without joint involvement - Glove, gowns, and eye protection

Contrast

Salter-Harris fracture that has the best prognosis

OSHA-designated personal protective equipments The use of single cane

- The cane is held on the stronger side of the body and the weight is shifted away from the weaker/painful limb - The cane is held on the side opposite the painful/injured side - A complex deformity involving bone, tendon, capsule, and soft tissue abnormality. - It has a similar appearance at birth to Talipes calcaneovalgus, but there is a lack of motion at the subtalar joint and ankle joint within 6 months after birth, a negative calcaneal inclination angle, and complete dislocation of the

Congenital vertical talus

42

H. Huynh NBPME Part II Rapid Review talonavicular joint - It cannot be reduced with serial casting The lumbrical tendon is located ____ to - Plantar the deep transverse metatarsal ligament An individual can functionally increase limb length by ___________________ - Supinating the subtalar joint - In closed kinetic chain supination of the STJ, the talus and tibia externally rotate, the calcaneus inverts, and the longitudinal arch of the foot is elevated, all of which result in a functionally longer limb - Normal value for the talocalcaneal angle is 20 to 40 degrees. - Become smaller with supination - Halothane - With the exception of halothane, all volatile anesthetics contain ether linkage. Volatile anesthetics with an ether linkage are less likely than halothane to produce cardiac dysrhythmias in the presence of exogenous epinephrine injection - A tendon translocation procedure - The tibialis anterior tendon remains attached at its insertion and is simply routed through a groove created in the navicular. Since its insertion has not changed, it is a translocation - Osteoarthritis

Talocalcaneal angle

General anesthetic that is most likely to sensitize the myocardium to catecholamines

Young procedure performed on the tibialis anterior

Heber dens nodes at distal IPSA is associated with ______

43

H. Huynh NBPME Part II Rapid Review Local anesthetic that is NOT recommended for use in children under 12 years of age - Bupivacaine - Bupivacaine is a long acting local anesthetic that carries an increased risk of cardiac and CNS toxicity - A viral infection of the skin characterized by the appearance of a few to numerous small, pearly, umbilicated papular epithelial lesions that contain numerous inclusion bodies known as molluscum bodies. - Venous stasis ulcerations

Molluscum contagiosum

Ulcer with red, granulating base

The most prominent physical finding of right-sided congestive heart failure In a flexor tendon transfer for hammer toe correction, the flexor tendon is inserted into the _______________

- Lower extremity dependent edema

- Dorsal aspect of the proximal phalanx - Surgically changing the insertion of the flexor digitorum longus tendon from the base of distal phalanx to the dorsal aspect of the proximal phalanx will correct the proximal interphalangeal joint contracture of the hammer toe by pulling the proximal phalanx into a more congruous position with the middle phalanx - An arthritic ankle and subtalar joint

An orthopedic shoe with a steel shank from heel to toe and a rocker bar would be appropriate for treating _________ Rocker sole

- Used for any type of pathologic or pathomechanical condition that limits normal movement of the ankle, metatarsal, or metatarsophalangeal joint - A dynamic lever that assists with the

44

H. Huynh NBPME Part II Rapid Review normal movement of the lower extremity during gait and prevents flexing of the sole of the shoe during all phases of gait Healing by first intention - Routine primary suturing - Demonstrated in the closure of an uncomplicated skin incision - Deep peroneal nerve

Motor innervation to the anterior compartment muscles of the lower leg Most common mechanism of injury in a lateral osteochondral talar dome fracture Hemolytic anemia Increase forefoot valgus deformity when casting is due to ___________

- Inversion, dorsiflexion

- Normochromic, normocytic anemia - Clinician pronated the STJ when locking the midtarsal joint - A pronated STJ would result in a pronation of the midtarsal joint, and forefoot valgus would increase - The relaxation of Achilles tendon reflexes is delayed in hypothyroidism - Audible bruit - Classic triad is hyperthyroidism with goiter, ophthalmopathy, and dermopathy - An oral anticoagulant - Has no effect on established thrombi, nor does it reverse ischemic tissue damage - Diazepam - Diazepam is insoluble in water and contains propylene glycol, a tissue

Hypothyroid neuropathy

Graves disease

Warfarin

Thrombophlebitis of the small veins frequently occurs with IV administration of ______________

45

H. Huynh NBPME Part II Rapid Review irritant that causes pain on injection and venous irritation Ventilation-to-chest-compression ratio in one-rescuer CPR - 2 ventilations:15 chest compressions - 2 ventilations for every 15 chest compressions - Generalized subperiosteal bone resorption is characteristic of hyperparathyroidism - Can affect bone in many ways, appears on radiographs as osteopenia. Excess parathyroid hormone results in an increase in bone metabolism, which may appear as microfractures, bone cysts, brown tumors, and pathologic fractures - One of the most common bacterial infections in people with HIV. - One study demonstrated the presence of these bacteria in 43 percent of people within 2 years of an HIV diagnosis. - This is a disseminated infection that thrives in the presence of a weakened immune system and low CD4+ counts - Without the disease - A cohort represents a designated group of individuals who are followed or traced over a period of time. - Epidermiology is the study of how a disease presents itself or occurs within a population. - Prospective studies examine patients rather than a review of the records - To study a disease and how it presents from one point in time forward, one

Hyperparathyroidism

Mycobacterium avium-intracellulare

In a prospective epideminologic study of a disease, the cohort originally selected consists of persons _____________

46

H. Huynh NBPME Part II Rapid Review looks at those who are free of disease Patellar reflex - Tests the femoral nerve, which innervates the quadriceps tendon. The nerve is made up of spinal roots L2,L3, and L4 - The use of oral contraceptive puts a woman at risk to develop deep vein thrombosis - Severe localized edema, erythema, serosanguineous drainage, induration, and pain on palpation 3 days postoperatively all indicate that a vessel was damaged, causing a hematoma - A medical emergency in AfricanAmerican patients, and the risks associated with this condition can be minimized by using a sickle cell trait screening test. - The initial treatment of choice for deep vein thrombosis - Measurement of the angle formed by the bisection of the second metatarsal and the line perpendicular to the lesser tarsus bisection. (normal 15 degree) - Metatarsus adductus is evaluated radiographically by utilizing second metatarsal and lesser tarsal axes - KOH or fungal culture and CBC - A KOH preparation and/or fungal culture should be done to ensure that a fungal infection exists before

Oral contraceptive

Hematoma

Sickle cell crisis

IV heparin

Metatarsus adductus angle

Tests that are important to perform when prescribe griseofulvin

47

H. Huynh NBPME Part II Rapid Review griseofulvin is prescribed due to the risk of side effects Crest pad - Used when one or several digits of a patients foot are in moderate contracture and are becoming irritated at the distal end of the digit. - Its purpose is to mechanically dorsiflex and extend the affected digits. - It creates an upward thrust and places weight bearing on the padded plantar aspect of the toes, alleviating painful lesions - Associated with lung cancer - The clubbing seen in lung cancer and certain other pulmonary diseases is generally considered to be a secondary effect of hypoxemia-induced local vasodilation and is thus an acquired deformity - The government agency responsible for chemical exposures - Reveals large bony erosions, which are caused by pressure necrosis of bone adjacent to a periarticular accumulation of monosodium urate crystals - The infective organism that most commonly causes septic arthritis in young adults. Commonly polyarticular in onset, it is often associated with skin lesions

True acquired clubbing of the fingernails

Occupational Safety and Health Administration (OSHA) Radiographic finding of chronic tophaceous gout

Neisseria gonorrhea

48

H. Huynh NBPME Part II Rapid Review Charcot joint - Caused by sensory neuropathy - Caused by the impairment of deep pain sensation or proprioception, which affects the joints normal protective reflexes. - It commonly occurs in patients with diabetes in joints formed by the tarsal bones - Complete excision of the lesser metatarsophalangeal joints. - Severe debilitating effects of rheumatoid arthritis commonly require this procedure

Hoffman-Clayton procedure

The usual mechanism of injury in football turf-toe injuries to the first metatarsophalangeal joint complex

- Hyperextension with compression - A plantar capsular-ligamentous sprain of the first metatarsophalangeal joint occurs when the hallux is hyperextended and compressed at the MPJ. This can be caused by the friction of artificial surfaces resulting in the fixation of the hallux, which causes hyperextension and jamming of the joint on the already dorsiflexed hallux. - Present in 40% of patients with RA, and marked flexion of the neck during anesthesia may cause severe neurological interruption. - The anesthesiologist should be consulted preoperatively for evaluation, and flexion-extension lateral cervical neck radiographs should be obtained for patients at risk

Atlantoaxial subluxation

49

H. Huynh NBPME Part II Rapid Review Acute septic joint disease - Acute septic joint disease produces a red, hot, swollen, and painful joint, which mimics the symptoms of gout. - Pulmonary angiogram

The gold standard for the diagnosis of pulmonary embolism Giant cell tumor

- Benign lesion, usually solitary and locally aggressive. - Classically appear with a radiolucent center and denser periphery. - Have eccentric and expansile, with a border that is well defined but not sclerotic, and have a soap bubble appearance

Q angle

- The Q angle is formed in the frontal plane by two line segments: - One segment from the tibial tubercle to the middle of the patella - One segment from the middle of the patella to the anterior superior iliac spine (ASIS) - Patellofemoral tracking problems and anterior knee pain are associated with an increased Q angle - An abnormal Q angle is most commonly associated with patellar malalignment - The combination of intravascular hyperviscosity due to a high red cell mass and a high platelet count with functionally abnormal platelets put patients with polycythermia vera at high risk for stroke, myocardial infarction, and venous thromboembolism

Polycythemia vera

50

H. Huynh NBPME Part II Rapid Review Long term steroid user preoperative management - Long term steroid usage suppresses the hypothalamic-pituitary axis (HPA) and increases the risk for hypotensive shock. - Preoperative supplementation with high-dose steroids will help avert the complication (increase steroid dose) - If an automated external defibrillator is to be used on a patient who has a medication patch, the patch should be removed prior to use of the defibrillator - Small burns to the skin may result if a defibrillator pad is placed on a medication patch - The chest compression should be 0.51.0 inch - Derived from a distinctive cell line of melanized cells - Arise from dendritic melanocytes - Benign nevus that is believed to form from nerves and not from the basal layer of the epidermis - A mild infectious disease of childhood cause by coxsackievirus - Two basic requirements for primary bone healing are: - An intact vascular supply to the bone - Stable rigid fixation of the fracture fragments

Automated external defibrillator

Chest compressions in infant cardiopulmonary resuscitation Blue nevus

Hand-foot-mouth disease

Primary bone healing

51

H. Huynh NBPME Part II Rapid Review Haglunds deformity - An increase in the Fowlers Philip angle of more than 75 degrees can be seen on a lateral radiograph - Positive parallel pitch lines or excessive bone on the calcaneus are also present. - These measurements are used to evaluate the extent of the deformity and to plan surgical correction - The degree to which it accurately provides a negative result for persons who are free of the disease - The most common cause of fogging on developed radiographic film - A segment of an individual states rights and are, therefore, determined by state law - An uncommon injury secondary to a crushing force to the physis through the epiphysis. - There is no visible fracture and no displacement - It is recommended that this injury be treated by placing the patient in a nonweight-bearing cast for 3 to 4 weeks, followed by rang-of-motion exercises to prevent premature fusion secondary to immobilization - The calcaneocuboid joint is prone to abduction in runners with pronation - This can then progress to subluxation, unless it is controlled with an orthotics device

Specificity

Light leak

Statues of limitations

Tx for Salter-Harris type V

Cuboid subluxation syndrome

52

H. Huynh NBPME Part II Rapid Review Candida albicans - A yeast that will reveal pseudohyphae on a KOH preparation - A sclerosing bone disorder characterized by numerous, small, symmetrical bone islands prominent in metaphyseal area - Diaphysis

Osteopoikilosis

In long bone, primary ossification occurs at _______ Gait of cerebral palsy patient

- Adducted, toe-walking, scissor-type gait - Develop the standards for quality of care by health systems

National Committee for Quality Assurance

Most common side effect of fentanyl, sufentanil, and thiopental

- Respiratory depression - They also cause cardiovascular depression and bradycardia, slowed gastric emptying, and antidiuretic hormone release. - The drug of choice for shock, angioedema, airway obstruction, bronchospasm, and urticaria in severe anaphylactic reactions. - Its action is via adrenergic effect - Mild to moderate eosinophilia is associated with drug allergy. - When drug-induced eosinophilia is suspected, termination of the offending agent generally terminates the problem

Epinephrine

Eosinophilia

53

H. Huynh NBPME Part II Rapid Review Wagner Classification - Wound classification 0 pre-ulcer area without open lesion 1 superficial ulcer (partial/full thickness) 2 Ulcer deep to tendon, capsule, bone 3 Grade 2 ulcer with abscess, OM, or joint sepsis 4 Localized gangrene 5 Global foot gangrene A = Neuropathic; B = Ischemic; C = Neuroischemic - Wound classification 0 No open lesion 1 Superficial wound 2 Tendon/ capsule 3 Bone/ Joint A = clean; B = non-ischemic infected; C = ischemic; D = ischemic infected - Wound classification I partial thickness ulcer extend through the epidermis and into, but not through the dermis II Full thickness ulcer extends to sub-cutaneous tissue only III Full thickness ulcer extends to lig/tendon/bone/joint IV Type III with abscess/OM V Type III with necrotic tissue in wound VI Type III with gangrene - Wound classification I non-blanchable erythema of intact skin II partial thickness skin loss

University of Texas Classification

Knighton Classification

NPUAP Pressure Ulcer Staging System

54

H. Huynh NBPME Part II Rapid Review involving epidermis and/or dermis III full thickness skin loss involving damage or necrosis of subcutaneous tissue that may extend down to but not through underlying fascia IV full thickness skin loss with extensive destruction, tissue necrosis, or damage to muscle, bone, or supportive structure Waldvogel and Lew Classification - OM classification Acute OM systemic clinical signs of infection Chronic OM subacute clinical signs of infection Contiguous/ direct extension spread of infection to bone from exogenous source or adjacent tissue Hematogenous spread infective agent reaches medullary canal of bone from vascular supply - OM classification Medullary infection of only the medullary canal Superficial infection of only superficial cortex Localized infection of only the cortex Diffuse infection of both cortex and medullary canal A = normal host; Bs = compromised host with systemic risk factors (DM); Bl = compromised host with local risk factors (smoking)

Cierny- Mader Classification

55

H. Huynh NBPME Part II Rapid Review Eichenholtz Classification - Charcot classification Stage 0 high risk pre-Charcot Stage 1 acute/ developmental Stage 2 coalescence Stage 3 reconstruction - Charcot classification Type 1 Lisfranc joint Type 2 Choparts joint and STJ Type 3A Ankle joint Type 3B Posterior calcaneus Type 4 Multiple combination Type 5 Forefoot - Open fracture classification Type I clean wound <1cm in diameter Type II wound 1-5 cm in diameter with minimal ST damage Type III wound >5cm in diameter with extensive ST damage; open fx > 8 hrs old IIIA: adequate ST coverage IIIB: extensive ST damage with periosteal stripping and massive contamination IIIC: arterial damage requiring primary repair - Foreign body classification I superficial/ cutaneous: usually visible without signs of infection II subcutaneous or articular without signs of infection IIIA subcutaneous or articular with signs of infection IIIB bone penetration without signs of infection

Brodsky Classification

Gustillo-Anderson Classification

Resnick Classification

56

H. Huynh NBPME Part II Rapid Review IV bone penetration with known OM Rosenthal Classification - Digital fracture classification Zone I injury occurs with damaged tissue completely distal to the distal aspect of the phalanx Zone II injury occurs with damaged tissue completely distal to the lunula Zone III injury occurs with damaged tissue completely distal to the most distal joint - Sesamoid trauma/ 1st MPJ trauma I dorsal dislocation of the proximal phalanx and sesamoids with intersesamoidal ligament intact IIA dorsal dislocation of the proximal phalanx with wide separation of the sesamoids without fx of either sesamoids IIB dorsal dislocation of the proximal phalanx with wide separation of the sesamoids with transverse fx of either sesamoids - 5th met base fracture I extra-articular at metaphysealdiaphyseal junction (true Jones fx) II intra-articular avulsion fx (1 or 2 fx lines) III extra-articular avulsion fx IV intra-articular, comminuted fx V extra-articular avulsion fx of the epiphysis - Lisfranc trauma Divergent first met moves medially while the other metatarsals move

Jahss Classification

Stewart Classification

Quenu and Kuss Classification

57

H. Huynh NBPME Part II Rapid Review laterally Convergent (homolateral) met 1-5 subluxed laterally Isolateral medial subluxation of 1st with other mets normal / lateral subluxation of mets 2-5 with 1st met remain normal Hardcastle/ Myerson Classification - Lisfranc trauma A total incongruity in any plane B partial incongruity B1 = 1st met goes medially B2 = lesser mets go laterally C divergent C1 = partial (only 1st and 2nd mets involved) C2 = total (all mets involved) - Navicular trauma I navicular tuberosity fx II dorsal lip avulsion fx IIIA navicular body fx with displacement IIIB navicular body fx without displacement IV stress fx of the navicular - Talar neck fracture I non-displace (13% AVN) II displaced fx with STJ dislocation (50% AVN) III displaced fx with STJ and AJ dislocation (95% AVN) IV displaced fx with STJ + AJ + TNJ dislocation (95% AVN)

Watson-Jones Classification

Hawkins Classification

58

H. Huynh NBPME Part II Rapid Review Berndt and Hardy Classification - Osteochondral defect (talar dome) I chondral depression II partial chondral fx seen on MRI III Nondisplaced complete osteochondral fx IV displaced complete osteochondral fx - Calcaneal fx (CT scan of posterior facet) I non-displaced articular fx II two-part posterior facet fx III three-part posterior facet fx IV four part/ comminuted fx - Calcaneal fx Type I A calcaneal tuberosity fx B sustentaculum tali fx C calcaneal ant process fx Type II A beak fx of calcaneal posterior tuberosity B avulsion fx of calcaneal posterior tuberosity Type III extra-articular calcaneal body fx (not involve STJ) Type IV intraarticular joint depression fx Type V intraarticular comminuted fx - Calcaneal fracture Tongue type fx Joint depression fx - Ankle fracture SAD vertical fx of medial mal I Lateral II Medial

Sanders Classification

Rowe Classification

Essex- Lopresti Classification

Lauge-Hansen Classification

59

H. Huynh NBPME Part II Rapid Review PAB short oblique fibular fx beginning at the level of syndesmosis (transverse on LAT view) I Medial II anterior, posterior III Lateral SER spiral fx of fibular beginning at AJ I anterior II lateral III posterior IV medial PER high fibular fx (Maisonneuver), oblique or spiral I medial II anterior III lateral IV posterior Danis-Weber Classification - Ankle fracture A below the AJ B at the AJ C above the AJ - Achilles tendon trauma I partial tear involving < 50% of tendon II complete tear with < 3 cm deficit III complete tear with 3-6 cm deficit IV complete tear with > 6 cm deficit - Lateral malleolar fx with ankle displacement - Trimalleolar fx

Kuwada Classification

Bosworths Fx

Cottons Fx

60

H. Huynh NBPME Part II Rapid Review Dupuytrens Fx Maisonneuves Fx Potts Fx Tillaux-Chaput Fx - Potts fx - Proximal fibular fx (fibular neck) - Bimalleolar fx - Avulsion fx of anterior inferior lateral tibial - Avulsion fx of anterior inferior medial fibular - Posterior malleolar fx - The rash of Lyme disease - Erythematous annular plaque with minimal central clearing on lateral leg - Pink, flesh-colored to violaceous nodules or diffuse brawny edema without the nodules - Infiltrative dermopathy most frequently appears symmetrically on the anterior tibia and dorsal of the feet in patients with Graves disease - Lacy, pink or violaceous areas on dorsal aspects of IPJs, elbows, patella, and medial malleoli pathognomic for dermatomyositis - A form of intraepidermal carcinoma which presents on sun-exposed areas as an indolent, erythematous, scaly plaque - Purple, pruritic, polygonal papules over the dorsum of foot and ankle

Wagstaffe Fx

Volkmann Fx Erythema chronicum migrans

Pretibial myxedema

Gottrons papules

Bowens disease

Lichen planus

61

H. Huynh NBPME Part II Rapid Review Atrophic blanche - Ulcers near the malleoli that heal with atrophic, white, stellate scars and treat with anticoagulants - Most common clinical findings of erythema induratum of Bazin - Clean, granulating centers with overhanging violaceous borders - Ulcerative lesions tends to favor the lower extremities, should not be debrided due to pathergy, and is associated with ulcerative colitis - Skin rash associated with Reiters disease - Asymptomatic hypopigmented scaling macules on lower legs - Fluoresce blue-green upon Woods lamp examination - Rough, scaly, erythematous plaques found on the anterior lower legs which seem to worsen during the summer - Most squamous cell carcinomas arise from this skin disorder - This skin manifestation may erupt in the unusual circumstance of a dermatophyte invasion into deeper layers of skin plaques with areas of alopecia - Squamous cell carcinoma that arises from a chronic wound

Painful ulcerative nodules on the calves

Pyoderma gangrenosum

Keratoderma blenorrhagicum

Pityriasis versicolor

Actinic keratosis

Majocchis granuloma

Marjolins ulcer

62

H. Huynh NBPME Part II Rapid Review Basal cell carcinoma - The most common non-melanoma skin cancer - Dermatological disorder that is characterized by reddish-brown papules in an annular or acriform distribution and may be seen on the legs of patients with DM - Skin disorder may cause blisters to spontaneously arise on the lower extremities of patients with diabetes - The most common skin finding on the lower extremities of patients with diabetes which is round to oval redbrown papules or plaques - Inflammation - Reparative/ Regenerative - Remodeling - Substrate/ Lag/ Inflammatory Stage (Day 1-4) - Proliferative/ Repair Phase (Day 3-21) - Remodeling/ Maturation Phase (Day 21+) - PASA + DASA < HAA - PASA and DASA is normal - Joint is subluxed (cross inside) - PASA + DASA = HAA - PASA and DASA is abnormal - Congruous joint

Granuloma annulare

Bullosa diabeticorum

Diabetic dermopathy

Phases of bone healing

Phases of wound healing

Positional deformity

Structural deformity

63

H. Huynh NBPME Part II Rapid Review Procedures for abnormal PASA - Distal osteotomy (head procedure) - Reverdin - Reverdin-Green - Reverdin-Laird - Distal osteotomy (head procedure) - Austin - Hohmann - Mitchell - Wilson - Distal metatarsal osteotomy - Reverdin Green - Hohmann - Shaft osteotomy - Ludloff - Mau - Base (proximal) osteotomy - Transverse closing base wedge - Crescentic - Juvara - Proximal V - Lapidus - Combined procedures - Logroscino - Proximal V osteotomy - Lapidus with Reverdin - Phalangeal osteotomy - Proximal Akin - Phalangeal osteotomy - Distal Akin - Angle produced at the intersection of the longitudinal bisection of the shafts

Procedures for IM = 12-16

Procedures for IM = 12-16 and abnormal PASA

Procedures for IM > 16

Procedures for IM > 16 and abnormal PASA

Procedures for abnormal DASA

Procedures for abnormal HAI

IM Angle

64

H. Huynh NBPME Part II Rapid Review of the 1st and 2nd metatarsal (normal 812 degree) Hallux abductus angle - Intersection of a line drawn through the long axis of the 1st metatarsal and the long axis of proximal phalanx (normal 0-15 degree) - Formed by a perpendicular line to the line of articular cartilage of the first metatarsal head and the bisection of the shaft of 1st metatarsal (normal 0-8 degree) - Formed by a perpendicular line to the line of articular cartilage of base of proximal phalanx and the bisection of the shaft of proximal phalanx (normal 08 degree) - Angle formed by the longitudinal bisection of the proximal phalanx with the longitudinal bisection of the distal phalanx (normal 0-10 degree) - Adductor hallucis is transected at its attachment to the lateral sesamoid and lateral base of proximal phalanx and rerouted over the met head and attached to medial capsule - Performed with HAV surgery to help realign the sesamoid apparatus under the metatarsal head - FDL is transected close to its insertion as the distal phalanx, split longitudinally to the base of the proximal phalanx, wrapped around the proximal phalanx

PASA

DASA

Hallux abductus interphalangeus angle (HAI)

Adductor tendon transfer

Flexor tendon transfer

65

H. Huynh NBPME Part II Rapid Review and sutured together - Aka: Girdlestone procedure Jones tenosuspension - EHL tendon is transected and rerouted medial to lateral through the head of 1st metatarsal and sewed back on itself - Distal stump of the EHL is then attached to EHB to maintain some extensor function of the hallux - Arthrodesis of IPJ to prevent overpowering of the EHL and hammering - Perform for pressure problems under 1st met head - Indications: flexible cavus foot, flexible PF 1st ray - EDL tendon slips are detached from their insertion combined and reattached to the 3rd cuneiform or base of 3rd met - EDB tendon are transected and reattached to the stump of the corresponding EDL tendon, the 4th and 5th longus slips are both attached to the 4th EDB slip - Releases the buckling force at MPJs and elevate FF - Indications: equinus with or without clawtoes - TA is split from its insertion up just proximal to the superior extensor retinaculum - The lateral fibers are passed through the peroneus tertius sheath and sutured to the tendon and attached to the cuboid - The procedure increases dorsiflexion of the foot and balances the force

Hibbs tenosuspension

Split tibialis anterior tendon transfer (STATT)

66

H. Huynh NBPME Part II Rapid Review laterally - Indications: flexible rearfoot varus, excessive supination, dorsiflextory weakness Peroneus longus tendon transfer - Peroneus longus is released at the level of the cuboid and transferred through the IM septum down the EDL sheath and inserted into the lesser tarsus or base of the 3rd met - Indications: drop foot, pes cavus - Transfer of all long extensor tendons to their respective metatarsal heads - Tibialis anterior tendon is transferred to the 3rd cuneiform through the EDL tendon sheath - Acts to reduce supination and increase dorsiflexion - Indications: drop foot, recurrent clubfoot, flexible forefoot equinus - The tibialis posterior tendon is transferred through the interosseous membrane and fixated to the 3rd cuneiform - This is an out of phase tendon transfer (stance swing) - Indication: drop foot, recurrent clubfoot - Posterior tubercle medial calcaneal artery (branch of posterior tibial artery) anastomosis with a branch from the peroneal artery to supply this area - Body supplied by the artery of the sinus tarsi (branch of the peroneal

Heyman procedure

Tibialis anterior tendon transfer (TATT)

Tibialis posterior tendon transfer

Blood supply to the talus

67

H. Huynh NBPME Part II Rapid Review artery) and the artery of the tarsal canal (branch of the posterior tibial artery) and there anastomosis as well as the deltoid branches - Head/Neck supplied by branches off the dorsalis pedis artery and the anastomosis that the dorsalis pedis makes with the deltoid branches (branch of posterior tibial artery) and the artery of the sinus tarsi Medial ankle structures from anterior to posterior - Tom, Dick, ANd Harry Tibialis posterior Flexor Digitorum longus Artery Nerve Flexor Hallucis longus - A HAND P Tibialis Anterior Extensor Hallucis longus Artery Nerve Extensor Digitorum longus Peroneus tertius - From Belladona plant, used as beauty aid to dilate pupils - Decreases secretions (tears, saliva, sweat) - Opioid - Used for induction of anesthesia - Has the advantage of no cardiac effects - Produces dissociative anesthesia, patient cooperates but with analgesia and amnesia

Dorsal foot structures from medial to lateral

Atropine

Fentanyl

Ketamine

68

H. Huynh NBPME Part II Rapid Review - Often used with BDZ due to emergence reaction, adverse hallucinations Midazolam Propofol (Diprivan) - Shortest acting IV BDZ - White IV liquid - Used to initiate and maintain anesthesia - Used for induction of anesthesia - A barbiturate - Also used as an anticonvulsant - Does not raise intracranial pressure - Contraindicated in porphyria - Blocks nicotinic receptors - Muscle relaxant used in anesthesia - Not attached by Ach-ase - Irreversible by drug - Short duration - May cause fasciculations - Blocks nicotinic receptors - Muscle relaxant used in anesthesia - Not attached by Ach-ase - Reversible by Neostigmine (better due to lack of CNS penetration) or Physostigmine - Longer duration - Does not cause fasciculations - Used by Indians for poison on their spears - Used for induction and maintenance of general anesthesia - An inhalation anesthesia - Hepatotoxic

Thiopental

Succinyl Choline

Tubocurarine

Halothane

69

H. Huynh NBPME Part II Rapid Review Isoflurane - An inhalation anesthesia - Decreases BP by vasodilation only, no cardiodepressant activity - An inhalation anesthesia - Decreases BP by vasodilation and cardiodepression - May cause seizures, especially if the patient has been hyperventilated - MAC is greater than 100% --not very potent - Least effects on cardiovascular system - Chronic exposure (dental office staff) may lead to pernicious anemia

Methoxyflurane Enflurane

Nitrous Oxide

Antidotes - Acetaminophen - Anticholinesterase - Benzodiazepine - Digoxin, Digitoxin - Methanol - Heparin - Iron - Lead - Methotrexate - Narcotics, Heroin - Tricyclic Antidepressants - Warfarin Virchows Triad

- Acetylcysteine - Atropine or Pralidoxine - Flumazenil (Romazicon) - Digoxin immune Fab - Ethanol - Protamine sulfate - Deferoxamine - Succimer - Leucovorin calcium - Naloxone - Physostigmine - Phytonadione (Vit K) - Three factors that are commonly associated with the formation of thrombi - Stasis (arrhythmias, MI, CHF, heart failure, immobilization, obesity, varicose veins, dehydration)

70

H. Huynh NBPME Part II Rapid Review - Blood vessel injury (trauma, Fx, IV) - Hypercoagulability (neoplasm, oral contraceptives, pregnancy, Sx, polycythemia) Stages of arterial occlusion - Intermittent claudication - Rest pain - Gangrene - > 55 mmHg healing - 45-55 mmHg range of uncertainty - < 45 mmHg no wound healing - At least 30 mmHg required for healing of a wound on the digits - Normal 1 - Calcified vessels > 1 - Intermittent claudication 0.5-0.8 - Rest pain and ulcers < 0.5 - Proliferation of the intima of small vessels has caused complete obliteration of lumen causing an insidious development of tissue ischemia - S/S - Foot is painful, cold, and numb - Skin is dry, scaly with dystrophic nails and poor hair growth - Atrophy of muscle and soft tissue - Edema is usually absent - Initial symptom is intermittent claudication - Rest pain may occur as disease advances - Ischemic feet may have ulcers which develop especially after local trauma - Severely ischemic feet develop

Toe pressure in diabetics

Ankle/Branchial Index (ABI)

Arteriosclerosis Obliterans (Arteriosclerotic occlusive disease)

71

H. Huynh NBPME Part II Rapid Review gangrene - Diagnosis: - Claudication - Rest pain - Gangrene - Dependent rubor and blanching on elevation - Treatment: - Daily walking to build up collateral circulation Thromboangiitis Obliterans (Buergers disease) - Inflammatory changes in small and medium sized arteries and veins. - Caused by some sort of hypersensitivity to tobacco. - Gradual onset beginning in the most distal vessels and progressing proximally, causing gangrene - Thrombosis of the superficial veins may also occur - S/S - Raynauds phenomenon is common - Intermittent claudication pain may occur, usually in the arch of the foot. Later, rest pain may occur - Frequently sympathetic nerve overactivity may occur; coldness, hyperhidrosis, cyanosis - < pulses - Inflammatory occlusions tend to be in the more distal arteries, resulting in circulatory insufficiency of the toes and fingers - Increased HLA-A9, HLA-B5 - Treatment: - Daily walking to build up collateral circulation - Stop smoking

72

H. Huynh NBPME Part II Rapid Review Monchebergs disease - A benign arteriosclerosis resulting in extensive deposits of calcium in the media layer of medium size arteries - This is a sclerotic, but not an occlusive disorder, and is usually an incidental finding of x-ray - Does not decrease blood flow but may decrease pulses due to lack of distensibility - Infrarenal aorta (80%) [20% in the iliac arteries, and 2% in the femoral and popliteal arteries] - Acute ischemia caused by an emboli thrown from somewhere else in the body. - Usually large emboli come from the heart, 40% of embolic obstructions are in the femoral artery and 20% are in the popliteal artery - S/S - Pain, pallor, paresthesia, paralysis, pulseless extremity - Sudden onset of severe pain, coldness, numbness and pallor - Pulses are absent distal to the obstruction - Diagnosis: angiography - Treatment: - Extremity is kept at low or below horizontal plane - Immediate embolectomy is the treatment of choice (best to perform within 4-6 hrs) - Delayed embolectomy (> 12 hrs) involves a high risk of acute respiratory distress syndrome or acute renal failure

Aneurysm mostly occur in _________

Arterial embolism

73

H. Huynh NBPME Part II Rapid Review - Anticoagulation rather than sx is indicated accepting urgent or elective amputation as the necessary lifesaving procedure Seddon classification - Nerve injury classification - Neurapraxia bruised nerve. Results in numbness that is reversible - Axonotmesis injury to axon that results in Wallerian degeneration. Will regenerate over several months as long as gap is not too big - Neurotmesis complete severance of the nerve resulting in irreversible numbness - Nerve injury classification - First degree a conduction deficit without axonal destruction - Second degree axon is severed without reaching the neural tube. Wallerian degeneration with regeneration. Regeneration is likely (axonotmesis) - Third degree degeneration of axon with destruction of fascicle with irregular regeneration - Fourth degree destruction of axon and fascicle and no destruction of nerve trunk, but a neuroma-in-continuity exists - Fifth degree complete loss, neuroma is likely and spontaneous recovery is rare - Tibial nerve - Saphenous nerve - Medial dorsal cutaneous nerve

Sunderlands classification

Ankle block

74

H. Huynh NBPME Part II Rapid Review - Deep peroneal nerve - Intermediate dorsal cutaneous nerve - Sural nerve Digital block (e.g. 3rd digit) - 5th and 6th dorsal digital proper nerve - 5th and 6th plantar digital proper nerve - 1st dorsal digital proper nerve - Deep peroneal nerve - 1st plantar digital proper nerve - 2nd plantar digital proper nerve - Saphenous nerve - Deep peroneal nerve - Medial dorsal cutaneous nerve - Medial plantar nerve - Lateral dorsal cutaneous nerve - 4th common dorsal digital nerve - Superficial branch of the lateral plantar nerve - 4th common plantar digital nerve - Sciatic nerve - Commonly affects 3rd common digital nerve - 3rd common digital nerve is located between and often distal to the 3rd and 4th metatarsal heads, plantar to the intermetatarsal ligament - S/S - Sullivans sign toes adjacent to the affected interspace splay apart on weight bearing - Lateral squeeze test point tenderness upon palpation of the plantar aspect between the metatarsal heads

Hallux block

Mayo block

Reverse Mayo block

Popliteal block Mortons neuroma

75

H. Huynh NBPME Part II Rapid Review while squeezing the metatarsal heads together - Mulders sign silent palpable click Joplins neuroma - A benign enlargement of the medial plantar digital proper nerve located on the plantar medial aspect of the first MPJ of the hallux - Tingling radiates distally when taping the tibial nerve - (+) in tarsal tunnel syndrome - Tingling radiates proximally when taping the tibial nerve - (+) in tarsal tunnel syndrome - A chronic inflammatory disease characterized by patchy demyelination of the CNS - S/S - Discrete motor, sensory, cerebellar, or visual attacks initially - Ocular palsy, fatigability/weakness, clumsiness, bladder dysfunction, spasticity - Charcots triad nystagmus, intention tremor, scanning speech (syllables are separated by pauses) - Excess heat may accentuate symptoms - Diagnosis: MRI, spinal tap, increase DTRs and (+) Babinski - Tx: corticosteroid - A devastating progressive degenerative disease of the UMNs and LMNs

Tinels sign

Valleixs sign

Multiple sclerosis

Amyotrophic lateral sclerosis (ALS, Lou Gehrigs disease)

76

H. Huynh NBPME Part II Rapid Review - Usually occurs when patients are in their 40s - Usually fatal within 2-5 years of onset - S/S - Wasting, weakness, cramps, stiffness, muscle twitching, spasticity of muscle groups, unexplained weight loss, or slurred speech - Musculoskeletal symptoms often beginning in the hands and spreading to the arms and legs - Increase DTRs - Mental status is usually preserved Guillian-Barre syndrome (Landrys paralysis) - Acutely progressive but self-limiting, acquired, inflammatory demyelinating polyneuropathy resulting in rapid weakness and paralysis - May causes life-threatening breathing difficulty - In severe forms, foot drop is noted - Symmetrical muscle weakness usually beginning in the legs and progressing to the arms - A hereditary, demyelinating, hypertrophic neuropathy of the peripheral nervous system - Characterized by slow progressive distal muscle atrophy resulting in foot drop - The earlier the onset, the poorer the prognosis - S/S - Weakness in the peroneal muscles - Pes cavus - Stork leg skinny legs due to peroneal atrophy

Charcot-Marie-Tooth Disease (peroneal muscular atrophy)

77

H. Huynh NBPME Part II Rapid Review - Inverted Champagne bottle legs - Foot drop (slapping gait) - Unsteady gait/tending to trip easily - Stocking-glove sensory loss - Decrease ankle DTRs - Later in the disease, the hallux becomes fixed and rigidly plantarflexed - Tx: AFO for foot drop Stages of Reflex Sympathetic Dystrophy (RSD) - Stage 1: Acute - Stage 2: Dystrophic - Stage 3: Atrophic - S1 - Due to closure of the atrioventricular valves (tricuspid and mitral) - Heard loudest at mitral and tricuspid areas - Use diaphragm of stethoscope - S2 - Due to closure of the Semilunar valves (aortic and pulmonic) - Heard loudest at aortic and pulmonic areas - Use diaphragm of stethoscope - S3 - Caused by rapid ventricular filling - Heard loudest at mitral area - May be normal in young people or due to CHF or mitral regurgitation - Use bell of stethoscope - S4 - Caused by forceful atrial ejection into a distended ventricle - Heard loudest at mitral area - May be normal (children, well-trained athletes) or due to HTN or aortic stenosis

Heart sounds

78

H. Huynh NBPME Part II Rapid Review - Use bell of stethoscope Aortic stenosis - Loudest at aortic area - Radiates to carotids and sometimes apex - Ejection click and S4 often heard at apex - Left lower sternal border - 3rd intercostals space left - Louder sitting forward and after exhalation - Heard loudest at apex (mitral area) - Left lateral decubitus position - Heard loudest at mitral area - Radiates to left axilla - Loudest at 2nd intercostals space below left clavicle - Continuous, machine murmur - Vasodilation - Increases local tissue metabolism - Increases capillary permeability - Increases collagen extensibility - Decreases joint stiffness - Arthritis - Stiff joints - Muscle spasms - Inflammation - Pain - Trauma (Heat can be used on subacute and chronic stages of injury, do not use heat during the first 48 hrs following injury or while active swelling occurs

Aortic regurgitation

Mitral stenosis

Mitral regurgitation

Patent ductus arteriosus

Physiological effects of therapeutic heat

Usages of therapeutic heat

79

H. Huynh NBPME Part II Rapid Review Superficial heat - Heat modalities where the temperature can be controlled - E.g. Whirlpool, contrast baths, paraffin wax - The heat source should be set around 45 degree C and treatment should run about 15-30 minutes - Contraindications: photosensitivity - Contraindications: metal implants, pacemakers - E.g. Ultrasound, Shortwave Diathermy, Microwave Diathermy

Deep heat

Physiological effects of therapeutic cold - Vasoconstriction - Decrease cellular metabolism (inflammation) - Decrease muscle spasm - Analgesia (numbling effect) - Decrease swelling Usages of therapeutic cold - Swelling - Pain - Muscle spasm - Inflammation - Trauma (Cold is used during the first 48 hrs following injury during acute swelling) - Static contraction - Muscle contraction that is not associated with joint motion or change in muscle length - Can be performed in a cast - Dynamic contraction - Muscle contraction with associated joint motion and change in muscle

Isometric contraction

Isotonic contraction

80

H. Huynh NBPME Part II Rapid Review length, either eccentric or concentric - Eccentric contraction muscle contraction in which the muscle lengthens while contracting. Contraction force of the muscle is against an overpowering weight resistance - Concentric contraction muscle contraction in which the muscle shortens as a result of the contraction. Where a weight is lifted with the force of muscle contraction Isokinetic contraction - Contraction at a constant velocity at all ranges of motion by using a machine with an accommodating resistance - Adjust height of crutches to allow 2 fingers width between axilla and axillary pad - Handpiece is adjusted to allow 30 degree of elbow flexion - Measure from ground to the greater trochanter for the length of the cane - Elbow should be flexed about 30 degree - Cane is held in and opposite affected foot - Absence of infection - Perfect hemostasis - Good dressing - Absence of motion - Hematoma blood develops under graft (most common) - Seroma serosanguineous fluid

Crutches

Canes

Regarding to skin grafts, what are the factor that a good take requires?

Reasons for graft failure

81

H. Huynh NBPME Part II Rapid Review develops under graft - Infection (second most common) - Shearing forces between graft and recipient site - Improper preparation of recipient site Skin features of arthropathies - Psoriatic arthritis - Reiters syndrome - Septic arthritis (esp N. gonorrhea) - Lyme arthritis - SLE Nasopharynx and ear features of arthropathies - Reiters syndrome - Gout Eye features of arthropathies - Reiters syndrome - Juvenile rheumatoid arthritis Gastrointestinal tract features of arthropathies - Crohns disease - Whipples disease - Ulcerative colitis Heart features of arthropathies - Lyme arthritis - Ankylosing spondylitis - SLE Nervous system features of arthropathies - Lyme arthritis

- Psoriasis - Keratoderma blennorrhagica - Rash - Erythema chronicum migrans - Butterfly rash, photosensitivity

- Stomatitis - Tophi

- Conjunctivitis - Iridocyolitis

- Diarrhea - Diarrhea - Diarrhea - Enlarged heart, arrhythmias - Aortic insufficiency - Pericarditis

- Stiff neck, headache

82

H. Huynh NBPME Part II Rapid Review Genitourinary system features of arthropathies - SLE - Gout Boutonniere deformity

- Nephritis - Kidney disease - Flexion of the PIPJ of the finger with hyperextension of the DIPJ in RA - Hyperextension of the PIPJ of the finger with flexion of the DIPJ in RA - A synovial fluid filled cyst which develops in the popliteal fossa. When ruptured symptoms mimic a DVT - Seen in RA - RA with associated splenomegalia. Patient presents with pigmented spots on the LE - The synovium develops into a vasculature granulation tissue that produces inflammatory agents and immunoglobin-producing lymphoreticularlike elements that destroy the articular cartilage - Seen in RA - Fever - Skin rash - Fusions of the cervical spine (especially C2-C4) - Polyarticular (40%) - Pauciarticular (40%) - Associated with Iridocyolitis - Stills disease (20%) - Associated with systemic

Swan neck deformity

Bakers cyst (popliteal cyst)

Feltys syndrome

Pannus transformation

In addition to arthritis, juvenile rheumatoid arthritis often associated with _____________

Three types of juvenile rheumatoid arthritis

83

H. Huynh NBPME Part II Rapid Review manifestations such as splenomegaly, generalized adenopathy Heberdens nodes - Bony protuberances at the margins and dorsal surface of the DIPJs in OA - Bony protuberances at the margins and dorsal surface of the PIPJs in OA - A chronic inflammatory arthritis that affects the sacroiliac joint and the spine - Bamboo spine - Arthritis - Urethritis - Conjunctivitis - Mucocutaneous lesions - Pencil in cup deformity - Whittling of the distal tufts of the phalanges Procainamide Hydralazine Chlorpromazine Isoniazid Penicillamine Griseofulvin

Bouchards nodes

Ankylosing spondylitis (MarieStrumpells disease)

Tetrad of Reiters syndrome

Radiographic features of psoriatic arthritis

Drugs that induce lupus

CREST

- Calcinosis cutis calcifications in the skin - Raynauds phenomenon - Esophageal dysfunction - Sclerodactyly localized scleroderma of the digits - Telangiectasia

84

H. Huynh NBPME Part II Rapid Review Polymyositis - A chronic progressive inflammatory disease of skeletal muscle - Characterized by symmetrical weakness of the limb girdles, neck, and pharynx, usually associated with pain and tenderness. - If associated with skin lesions dermatomyositis - Skin manifestations include: - Violaceous (heliotrope) inflammatory changes of the eyelids and periorbital area - Erythema of the neck, face, and upper trunk - Gottrons sign flat-topped violaceous papules over the dorsal aspect of the knuckles - Chronic autoimmune inflammatory disorder of the exocrine glands - Resulting in decreased secretions in many areas of the body - Keratoconjunctivitis sicca (dry eyes) - Xerostomia (dry mouth) - Crohns disease and ulcerative colitis - Commonly involves the knees, ankles, elbows, and wrists - Decreases

Sjogrens syndrome

Arthritis associated with inflammatory bowel disease

Bohlers angle ________ in calcaneal fracture Gissanes angle _________in calcaneal fracture Calcaneal inclination angle

- Increases

- Normal 20-25 - Value does not change with pronation and supination

85

H. Huynh NBPME Part II Rapid Review - Decreased in pes planus - Increased in pes cavus Kites Angle - Talocalcaneal angle - Normal 20-40 - Increase in pronation - Decrease in supination - Stance phase (62%) - Contact (27%) heel strike to forefoot loading - Midstance (40%) forefoot loading to heel off - Propulsive (33%) heel off to toe off - Swing phase (38%) - 5mm discrepancy - Walk with shoulder drop - During stance, the foot of the longer leg is usually pronated and the shorter leg is supinated - Describes the movement of a body part that is NWB - Open kinetic chain pronation: - abduction, eversion, and dorsiflexion of foot - Describes movement of a bone around a joint in a body part that is WB - Closed kinetic chain pronation can be described as: - Talar plantarflexion, adduction in ankle joint, and calcaneal eversion

Gait cycle

Limb length discrepancy (LLD)

Open kinetic chain

Closed kinetic chain

86

H. Huynh NBPME Part II Rapid Review Flexor stabilization - Occurs in flat feet with excessive pronation - STJ pronation allows hypermobility and unlocking of the midtarsal joint leading to hypermobility of the forefoot - The flexors fire earlier and longer than normal in an attempt to stabilize the forefoot - Flexors overpower the interosseous muscles and cause digital hammering or clawing - Adductovarus of the fourth and fifth toes because the quadratus plantae loses its mechanical advantage - Occurs with weak triceps surae - The deep posterior leg and lateral leg muscles try to compensate for lack of plantarflexion - Create a high arched supinated foot and contract the digits - NO adductovarus deformity - Swing phase - Extensors gain mechanical advantage over the lumbricales and contract the MPJs - NO adductovarus deformity

Flexor substitution

Extensor substitution

87

H. Huynh NBPME Part II Rapid Review Spastic gait - Manifested by internal rotation and adduction of the entire limb with hip/knee/ankle in marked flexion - Seen with CP, familial spastic diplegia, paraplegia, and hemiplegia

Dyskinetic gait

- Characterized by motion involving considerable effort, often with deliberated almost concentrated steps - Seen with CP, Huntingtons chorea, and dystonia muscular deformities - Characterized by a marked instability during single limb stance with an alternating wide/narrow base during double support - During swing phase, the limb will swing widely and cross the midline - Seen with MS, tabes dorsalis, diabetic polyneuropathy, Fredrichs ataxia

Ataxic gait

88

H. Huynh NBPME Part II Rapid Review

Waddling gait

- A laboring gait exhibiting difficulty with balance - Proximal pelvic instability leading to lumbar lordosis - May see an associated Equinovarus foot type - Seen with muscular dystrophies, spinal muscular atrophy, and congenital dislocated hip

89

H. Huynh NBPME Part II Rapid Review Steppage gait - Gait exhibits a swing phase foot drop - Seen with Charcot-Marie-Tooth, polio, Guillain-Barre syndrome, CVA, paralytic drop foot, and fascioscapulohumeral dystrophy

Vaulting gait

- Gait changes include a high step rate, increased lateral trunk movement, scissoring and instability from step to step suggesting loss of balance - Seen with myotonic dystrophy - Gait exhibiting a swing phase ankle plantarflexion with no heel contact - Seen with CP, CMT, MD, spinal muscular atrophy, schizophrenia, osseous block of the ankle, and habitual toe walking - Shuffling gait with loss of reciprocal arm swing, decreased velocity, decreased stride length, and increase step rate - Seen with Parkinsons disease

Equinus gait

Fenistrating gait

90

H. Huynh NBPME Part II Rapid Review Trendelenberg gait - Stance phase of each step leads to a contralateral tilt of the pelvis with a deviation of the spine to the affected side - Seen with dislocated hip or weakness of the gluteus medius muscle - Counteracting or avoiding pain, walking with a limp to lessen pain. - Control the contrast of a film - Increasing the kilovolts produces a more penetrating x-ray but poor contrast - Increasing the kVp results in less exposure to the patient but a lower quality picture - Controls the intensity of the x-rays emitted from the x-ray tube - The most important component controlling density - Increasing mA produces a brighter radiograph but little change in overall contrast - To reduce radiation exposure decrease mA - To achieve maximum fidelity (true size/shape of original object), the distance of the object to the film must be kept to a minimum - A small focal spot gives better detail - The absorbed radiation involving the interaction with the inner electron shell - Beneficial to the image and increases the exposure to the patient

Antalgic gait

Kilovolts (kVp)

Milliamperage (mA)

Distance

Photoelectric effect

91

H. Huynh NBPME Part II Rapid Review - Increases at lower kVp Hard x-rays - Produced by increased kVp - Have a short wavelength, high frequency, increased penetration - Less dangerous to patient - Produced by decreased kVp - Have long wavelength, low frequency, low penetration - More dangerous to patient

Soft x-rays

Relative densities of different materials - Cortex (highest to lowest) - Cancellous - Muscle - Nerve - Tendon - Ligament - SubQ - Fat - Air The x-ray view which demonstrate any anterior ankle impingement (osseous equinus) - Stress lateral or stress dorsiflexion (position patient for a lateral but then have the patient flex knees and maximally dorsiflex ankle) - Fat lights up - Bone marrow, nerves, lipomas - Water lights up - Blood, inflammation, edema, tumors - OM - Trauma/ inflammatory arthritis - Stress fracture as early as 7 hrs after insult - Tumors

T1 weighted image

T2 weighted image

Podiatric indications for bone scan

92

H. Huynh NBPME Part II Rapid Review - Nonspecific pain 4 phase bone scan - 1st phase radionuclide angiogram (blood flow phase) - Images taken 1-3 seconds apart immediately following injection - Provides information about the relative blood supply to the extremity - 2nd phase blood pooling images - Imaging taken 5-10 minutes following injection - Quantifies relative hyperemia or ischemia - 3rd phase delayed image (boneimaging phase) - Imaging at 3-4 hrs following injection - If cellulitis there should be a flushing and cleaning returning toward normal density - If OM Tc-99 will incorporate into the bone and show increased density - 4th phase - Used in the diagnosis if OM - Shows greater bone activity and less soft tissue activity - Images are taken at 24 hrs - Combining technetium (bone imaging radionuclide) and gallium (inflammatory imaging nuclide) - Technetium should be given first, because it has a shorter life, followed in 24-48 hrs by gallium - Results referred to phase 3 as either (+) or (-) - Tc tells if bone is involved and Ga tells if WBCs are involved

Combined Tc and Ga bone scan

93

H. Huynh NBPME Part II Rapid Review Ga(+) and Tc(+) - OM - Ga(+) indicates that infection is present and Tc(+) indicates that bone is involved - Cellulitis - Ga(+) indicates that soft tissue infection is present and Tc(-) indicates that bone is not involved - Osteoarthropathy/Stress fracture/ Chronic OM - Ga(-) indicates that there is no infection and Tc(+) indicates that bone is involved - Breast - Lung - Prostate - Kidney - Multiple myeloma - Osteochondroma

Ga(+) and Tc(-)

Ga(-) and Tc(+)

The most common sources of metastatic bone tumors

The most common benign primary bone tumor The most common malignant primary bone tumor Bone tumor in the epiphysis Bone tumors in the metaphysis

- Multiple myeloma

- Chondroblastoma - Osteosarcoma - Enchondroma - Unicameral bone cyst - Chondrosarcoma - Aneurysmal bone cyst - Chondromyxoid fibroma - Fibrosarcoma - Nonossifying fibroma

94

H. Huynh NBPME Part II Rapid Review Bone tumors in the diaphysis - Ewing sarcoma - Sarcoma - Osteochondroma - Ossifying fibroma - Giant cell

Bone tumor extends from the metaphysis through the physis and end up in the epiphysis Osteoblastoma (giant osteoid osteoma, osteogenic fibroma)

- Rapidly growing benign bone forming tumor that can become malignant - Most commonly seen in the spine, skull, and the diaphysis of long bones - Mild pain worse at night not relieved by aspirin - Well circumscribed, expansile, osteolytic lesion (>1cm) with areas of calcifications and cortical thinning - Benign bone forming tumor closely related to fibrous dysplasia - Intracortical osteolytic lesion with a ground glass appearance that may be a single confluent region or a multiple elongated bubbly area clearly delineated by a band of sclerosis - Benign bone tumor with ground glass matrix that can convert to malignant - Associated with McCune-Albrights syndrome - Leading to varus deformity of the femur the shepherds crook deformity - Pathological fractures and rickets-like bowing occur because normal osseous tissue is replaced by fibrous tissue weakening the bones

Ossifying fibroma

Fibrous dysplasia

95

H. Huynh NBPME Part II Rapid Review Osteosarcoma (Osteogenic sarcoma) - Malignant bone forming tumor - Usually occurs in teenagers during rapid growth spurts or in patients over 40 who have a preexisting condition mot notably Pagets disease - Most common site for metastases: lung - Benign cartilage forming tumor - An expansile, oval, lucent lesion - Found in proximal phalanges - Can convert to malignant lesion (chondrosarcoma) - Enchondromatosis - Olliers disease - Multiple chondromas - May become malignant - Maffuccis syndrome - Multiple chondromas associated with soft tissue hemangiomas - May become malignant - Benign cartilage forming tumor - Usually in the talus or calcaneus - Well defined round or oval osteolytic region, eccentrically or centrally located that may have a thin sclerotic border - chicken wire calcification - May mimic an aneurysmal bone cyst - Benign cartilage forming tumor - Radiolucent lesions, some appearing lobulated or bubbly with well-developed sclerotic borders - Most common benign tumor - Can convert to malignant - Benign cartilage covered osseous protuberance

Enchondroma

Chondroblastoma

Chondromyxoid fibroma

Osteochondroma

96

H. Huynh NBPME Part II Rapid Review - Normal trabeculation pointing away from the joint (trabecular pattern usually run perpendicular to the axis of long bone) - Can resemble a miniature physeal region - Most commonly seen subungually in the foot - Dupuytrens exostosis subungual osteochondroma Chondrosarcoma - Malignant cartilage forming tumor - Malignant bone tumor in which patients with Olliers disease are at risk for - Patient profile: male in his fifties - Benign connective tissue tumors - Greater than 4cm while fibrous cortical defect is less than 4 cm - Malignant connective tissue tumors - Osteolytic foci with a geographic, motheaten or permeative pattern of bone destruction. - Occurs de nova or secondary to Pagets disease, osteonecrosis, chronic OM, radiation or other undifferentiated neoplasm - Aggressive tumor that has a tendency to reoccur - Plantar fibromatosis histoligically may resemble fibrosarcoma - Benign fibrohistocytic tumors composed of connective tissue, stromal cells and giant cells - May be associated with Pagets disease - Eccentric osteolytic lesion extending

Nonossifying fibroma

Fibrosarcoma

Giant cell tumor

97

H. Huynh NBPME Part II Rapid Review to the subchondral bone - Can convert to malignant - soap bubble matrix Unicameral bone cyst (solitary bone cyst, simple bone cyst) - Benign tumors of unknown origin - Fallen fragment sign with a pathologic fracture, a bone fragment falls into the cyst and migrates to the dependent position - Lesions in patients > 20 y/o, occur in the pelvis or calcaneus - Calcaneal lesions are well-defined, radiolucent lesions usually occurring at the neutral triangle - Benign blood fill cyst - Rapidly growing painful lesion - An eccentric, osteolytic expansive lesion which may extend out into the soft tissue - Trauma is the possible etiology - Soap bubble matrix - Malignant tumor - Onion skin changes in the periosteum - Symptoms include pain, swelling, fever, weight loss, and leukocytosis - Usually occurs between the ages of 720 y/o - Aneurysmal bone cyst - Giant cell tumor - Chondromyxoid fibroma - Osteochondroma - Enchondroma - Giant cell - Fibrous dysplasia

Aneurysmal bone cyst

Ewings sarcoma

Three benign bone tumors that can have a soap bubble matrix

Four benign bone tumors that can convert to a malignant one

98

H. Huynh NBPME Part II Rapid Review Two most common bones of the foot affected by primary bone tumors Tumors with geographic bone destruction - Calcaneus - Metatarsals - Non-ossifying fibroma - Chondromyxoid fibroma - Eosinophilic granuloma - Myeloma - Metastases - Lymphoma - Ewings sarcoma - Lymphoma, leukemia - Ewings sarcoma - Myeloma - Osteomyelitis - Neuroblastoma - Geographic - Motheaten - Permeative - None - Solid (chronic osteomyelitis) - Onion-skin (Ewings sarcoma) - Sunburst (osteosarcoma) - Codmans triangle (Ewings sarcoma) - Solid - Lamellated - Sunburst - Codmans - Fluffy, cotton-like or cloud-like densities - E.g. osteosarcoma

Tumors with motheaten bone destruction

Tumors with permeative bone destruction (round cell lesions)

Patterns of bone destruction from least malignant to most malignant

Benign periosteal reactions

Aggressive/ Malignant periosteal reactions

Periosteal reactions from least malignant to most malignant

Osteoblastic tumor matrix

99

H. Huynh NBPME Part II Rapid Review Cartilaginous tumor matrix - Comma-shaped, punctuate, annular, popcorn-like - Enchondroma, chondrosarcoma, chondromyxoid fibroma - Multiple myeloma - Mets - Aneurysmal bone cyst - Fibrous dysplasia - Brown tumor - Enchondroma - Lymphoma

Expansile lesions of bone

Clues by location of lesions in the transverse plane - Central - Eccentric - Cortical - Parosteal

- Enchondroma - Giant cell, osteosarcoma, chondromyxoid fibroma - Non-ossifying fibroma, osteoid osteoma - Parosteal osteosarcoma, osteochondroma

Clues by location of lesions in the longitudinal plane - Epiphyseal - Metaphyseal - Diaphyseal

- Giant cell, chondroblastoma - Osteomyelitis, osteosarcoma, chondrosarcoma - Ewings sarcoma, myeloma, neuroblastoma, lymphoma, aneurysmal bone cyst, enchondroma

Characteristic locations of tumor - Simple bone cyst - Chondroblastoma - Giant cell tumor

- Proximal humerus - Epiphyses - Epiphyses

100

H. Huynh NBPME Part II Rapid Review Four stages of bone healing - Inflammation peaks at 48 hrs and subsides about a week - Soft callus begin several days after injury and persists for about 1-2 months - Hard callus occurs at around 3-4 months if a soft callus is successful in connecting the fracture - Remodeling lasts for several years

The final bone morphology is determined - Wolffs law by ___________ Primary bone healing (membranous) - Haversian remodeling consists of simultaneous remodeling and direct formation of new bone - Little or no callus formation - Occurs when there is good opposition and NO motion at the fracture or osteotomy site - Desirable method of healing - Involves the formation of cartilaginous and fibrous tissue intermediates that a later replaced by bone - Involves a callus formation - Occurs when there is motion at the fracture/osteotomy site replaced by bone - Less desirable method of healing - From ones own body or identical twin - Store in closed container covered with a moistened saline sponge, without immersion - Fresh water is present - Cellular bone matrix - Osteoinductive properties - Allows creeping substitution

Secondary bone healing (enchondral)

Autograft

101

H. Huynh NBPME Part II Rapid Review - Limited amount - Faster healing - Requires a donor site Allograft (homograft) - Dead bone from same species - Freeze dried/ devoid of water - Noncellular bone matrix - No osteoinductive properties - Allows creeping substitution - Unlimited amount - Slower healing - Bone bank bone - Bone from different species - Hydroxyapatite, tricalcium phosphate, type I collagen, marine coral - Osteogenesis - The formation of bone - Creeping substitution - The process by which most cellular elements in grafts die and are slowly replaced by viable bone - Osteoconduction - The scaffolding effect of bone graft that acts as a conduit for migration of viable cells - The matrix that allows creeping substitution to occur - Osteoinduction - The presence of bone morpheogenic protein inductor substance that causes nonosseous tissue to become osteogenic - Non-immunologic mechanism - A single exposure causes a reaction - E.g. detergents, fiberglass

Xenograft (heterograft) Synthetic grafts

Phases of graft healing

Irritation contact dermatitis

102

H. Huynh NBPME Part II Rapid Review Allergic contact dermatitis - Acquired immunologic response - First contact causes no reaction, but the exposure sensitizes the skin to future exposures - E.g. Ivy poison - Dermatitis resulting from hereditary predisposition to a lowered cutaneous threshold to puritis - There is usually a positive family history of allergic rhinitis, hay fever, asthma, or migraine headache - Often symmetrical - An allergic reaction resulting in transient puritic wheals or small erythematous papules that erupt in minutes to hours and disappear usually within 24 hours or less - A circumscribed area of lichenification resulting from repeated physical trauma (rubbing/scratching) - Dermatitis of the lower leg related to PVD - Usually found just proximal to medial malleolus - Often associated with brown reticulated hemosiderin hyperpigmentation - A rare disease frequently associated with GI diseases (ulcerative colitis, Crohns disease) - Consists of large ulcers with characteristic purple overhanging edges which develop rapidly from pustules and

Atopic dermatitis

Urticaria

Lichen simplex chronicum (neurodermatitis)

Stasis dermatitis

Pyoderma Gangrenosum

103

H. Huynh NBPME Part II Rapid Review tender nodules - Occur particularly on lower legs, abdominal, and face A common contagious superficial skin infection Pitted keratolysis - Impetigo

- Superficial pitting in the stratum corneum on the soles of the feet giving rise to a motheaten appearance - A bacterial infection affecting the intertriginous areas of the body (between toes, groin, and axillae) - Often a secondary infection as a result of tinea - Woods lamp will cause the area to fluoresce coral red - An acute, severe, rapidly spreading skin infection (more specifically an infection of the connective tissue just beneath the skin) - Most common pathogens are group A streptococcus pyogenes and staph. aureus - Acute superficial form of cellulitis involving the dermal lymphatics

Erythrasma

Cellulitis

Erysipelas

Three most common cause of tinea pedis - Most common - Second most common - Third most common Molluscum Contagiosum

- Trichophyton rubrum (50%) - Trichophyton mentagrophytes (45%) - Epidermophyton floccosum (5-10%) - Contagious viral infection of the skin - Sexually transmitted in adults - Poxvirus is the cause

104

H. Huynh NBPME Part II Rapid Review Psoriasis - A common, chronic inflammatory dermatitis - Extensor surfaces of the extremities - Auspitz phenomenon removal of silvery scales results in pinpoint bleeding - Pitting of fingers and toenails - A recurrent, benign, puritic, inflammatory eruption of the skin, associated with oral lesions in about half of patients - A benign skin lesion composed of small blood vessels - Rapidly developing bright-red or violacenous or brown-black nodule - Lesions bleed easily when only slightly touched - Commonly occurs at nail margins and grooves, especially in conjunction with ingrown toenails - Tx: excise lesion - A benign, extremely painful, vascular tumor usually found subungual - Popoff tumor - Classic - African - Immuno-suppressed patient - AIDS-related - rodent ulcer - Most common type of skin cancer (malignant)

Lichen planus

Pyogenic granuloma

Glomus tumor

Four types of Kaposis sarcoma

Basal cell carcinoma

105

H. Huynh NBPME Part II Rapid Review Squamous cell carcinoma - A malignant tumor of epithelial keratinocytes - High incident of metastasis - Most frequently associated with chronic venous ulcers - Squamous cell carcinoma in situ - A superficial variant of squamous cell carcinoma that resembles a localized patch of psoriasis, dermatitis, or tinea - Superficial spreading melanoma - Nodular melanoma - Lentigo maligna melanoma

Bowens disease

Most common type of melanoma Melanoma with worse prognosis Slowest growing and least likely to metastasize melanoma Melanoma that is most commonly seen in Blacks and on the soles of the feet

- Acral lentiginous melanoma

Two classification systems for melanoma - Clarks classification - Breslows classification Clarks classification based on ______ Breslows classification based on _____ Cutaneous manifestations of diabetes - Depth of invasion - Thickness of tumor - Necrobiosis Lipoidica Diabeticorum - Xanthoma Diabeticorum - Bullous Diabeticorum - Diabetic Dermopathy - Diabetes

Leading cause of Charcot foot

106

H. Huynh NBPME Part II Rapid Review Three types of wound closure - Primary closure - Wound is immediately sutured; must be a clean wound. Leaving a pleasing linear scar - Infected or dirty wounds are left open and allowed to granulate in from the bottom up; leaves a less pleasing scar - Infected or dirty wound is left open until immediate treat of infection has passed (days to weeks) and then later the wound is closed primarily with sutures - Branch of lateral plantar nerve - Also called nerve to abductor digiti quinti muscle - Compensated rearfoot varus - Compensated forefoot varus - Compensated forefoot valgus - Plantarflexed 1st ray - Rarely occurs without coexisting involvement of the lymphoid organs - There is usually a known history of leukemia or other lymphoid organ disease - Appear in the skin as mycosis fungoides Lipoma

- Secondary closure

- Delayed primary closure

Baxter nerve

Most common causes of Haglands deformity

Malignant lymphoma

Soft tissue mass that associated with Gardners syndrome and neurofibromatosis Liposarcoma Two types of Schwannoma

- Not derived from pre-existing lipoma - Neurofibroma - Neurilemoma

107

H. Huynh NBPME Part II Rapid Review Neurofibroma - Recklinghausens disease when multiple cutaneous lesions are noted with caf au lait spots - Locates in central of the peripheral nerve - Present as a solitary, painless, fusiform, round, or oval mass that is sharply circumscribed and encapsulated - Develops along the coarse of a digital nerves - Tend to favor the flexor surfaces of the extremity - Common skin cyst that occurs secondary to traumatic implantation of the epidermis into the dermis, the epidermal cells continue to grow and accumulate keratin within the cyst - One of the most common lesions found in the foot - Found on the plantar aspect of the foot - Foul smelling - An epidermal inclusion cyst at the MPJ caused by trauma from the vamp portion of the shoe - Giant cell tumor involving the joint space - Synovial sarcoma

Neurilemoma

Epidermal inclusion cyst

Vamps disease

Pigmented villonodular synovitis

Most common malignant soft tissue lesion in the foot ASA Surgical Risk Classification -I - II

- Healthy patient - Patient with mild systemic disease (e.g.

108

H. Huynh NBPME Part II Rapid Review - III - IV -V - IV essential HTN, NIDDM) - Patient with severe systemic disease that limits activity (e.g. angina, COPD) - Patient with incapacitating systemic disease that is a constant treat to life - Moribund patient not expected to survive 24 hrs with or without surgery - Patient declared legally brain dead and awaiting organ harvesting - Creatine kinase

Increase level of ________ may indicate a threat of developing malignant hyperthermia Sequence of inserting a 2.7mm cortical bone screw

- 2.0mm thread hole - 2.7mm glide hole - Countersink - Depth gauge - 2.7mm tap - Flush - Insert 2.7mm screw - Allows compression across a fracture/ osteotomy site as screws are tightened due to plate screw hole shape - Allows stress to be transmitted through the plate so as to avoid the fracture/ osteotomy site - Used to maintain separation of bone to protect a bone graft from being crushed - The plate is placed on the tension side of a fracture/ osteotomy will present distraction on that side, while causing a corresponding compression on the other side

Dynamic compression plate

Neutralization plate

Buttressing plate

Tension-Band plate

109

H. Huynh NBPME Part II Rapid Review Swing phase muscles - Tibialis anterior - EHL - EDL - Peroneus tertius - Gastrocnemius - Soleus - FHL - FDL - Peroneus longus - Peroneus brevis - All intrinsics - Positive when a palpable click is felt as the femoral head is made to enter the acetabulum - Relocate the dislocated hip - Dislocate an unstable hip - The dislocatable hip becomes displaced with a palpable clunk as the head slips over the posterior aspect of the acetabulum - Look for asymmetry of thigh and gluteal folds - There will be more folds on the dislocated side - A dislocated hip result in a lower knee position on the affected side - A dislocated hip will have limitation of abduction on the affected side - A line connecting the most inferior portion of the acetabulum on both sides

Stance phase muscles

Ortolanis sign

Barlows sign

Anchors sign

Galezzis sign (Allis sign)

Abduction test

Hilgenreiners line

110

H. Huynh NBPME Part II Rapid Review Ombredannes line - A line perpendicular to Hilgenreiners line at the outer most aspect of the acetabulum - After drawing the Hilgenreiners line and Ombredannes lines, the normal position of the developing femoral head should be in the lower medial quadrant - A dislocated hip will show at least part of the femoral head in the outer upper quadrant - Should be between 27-30 degree at birth and decreases to 20 degree by age two. - An angle greater than 30 degree indicates a dislocated hip - Ankle equinus - Hindfoot varus - Forefoot adduction - Decrease Kites angle - Decrease calcaneal inclination angle - Talar head/neck is adducted and plantarflexed relative to talar body

Quadrant system

Acetabular index

Triplanar deformity of club foot (Talipes Equinovarus)

Radiographic evaluation of club foot

Posterior soft tissue release in club foot - Reflect the origin of abductor hallucis treatment and plantar fascia - Z-plasty of Achilles tendon - Release of posterior, medial, and lateral ankle joint - Release of posterior, medial, and lateral STJ the posterior talofibular and calcaneofibular ligaments are severed.

111

H. Huynh NBPME Part II Rapid Review Medial soft tissue release in club foot treatment (talonavicular and medial STJ release) - Z-plasty of the posterior tibial tendon - Release of the talonavicular joint the spring ligament and Henrys knot are severed - Release entire medial STJ including the superficial deltoid ligament - Release of the interosseous talocalcaneal ligament - Release of the bifurcate ligament - Release of the lateral STJ - Performed along with soft tissue releases after the child reaches at least 1 year old - Lichtblau anterior calcaneal osteotomy - Evans cuboid-calcaneal osteotomy - Ganleys closing abductory cuboid osteotomy - Z foot, serpentine foot, compensated metaductus - Adducted forefoot, normal midfoot, and a valgus hindfoot - Increased calcaneocuboid angle - High arched foot - Primarily a Sagittal plane deformity - Often the first manifestation of many neuromuscular disorders - bullet hole sinus tarsi - Increase calcaneal inclination angle - Increase angle of Meary - Increase angle of Hibbs

Lateral soft tissue release in club foot treatment (performed through the STJ in the single medial incision approach)

Osseous procedures for club foot treatment

Skewfoot

Pes cavus

Radiological evaluation of pes cavus

112

H. Huynh NBPME Part II Rapid Review Soft tissue procedures for flexible pes cavus - Plantar fascial release

- Tendon transfers

- Steindler stripping plantar fascia and the long plantar ligament is released; abductor hallucis, FDB, and abductor digiti quinti are stripped from the periosteum of the calcaneus - Jones tenosuspension - Heyman procedure transfer of all four extensor tendons to their respective metatarsal heads - Hibbs procedure - STATT - Peroneus longus tendon transfer - Tibialis posterior tendon transfer - Peroneal anastomosis at the lateral ankle, the peroneus longus is anastomosed to the peroneus brevis. This decreases plantarflexion of the 1st metatarsal and increases eversion forces of the foot - Peroneus longus and tibialis posterior tendon transfer to calcaneus

Osseous procedures for rigid pes cavus - Cole

- Japas - DuVries - Dwyer - McElvenny-Caldwell - DFWO

- Dorsiflexory wedge osteotomy through the navicular-cuneiform joint and cuboid bone - Midtarsal V-osteotomy; no bone is excised - Dorsiflexory fusion through the MTJ - Lateral closing wedge or an opening medial wedge - Dorsiflexory fusion of the 1st metatarsal-1st cuneiform joint - Dorsiflexory osteotomy of the 1st metatarsal or all metatarsals

113

H. Huynh NBPME Part II Rapid Review - Jahss (truncated tarsometatarsals wedge osteotomy) The most common cause of peroneal spastic flatfoot Common types of tarsal coalitions - Dorsiflexory wedge osteotomy across the tarsometatarsals articulations - Tarsal coalition

- Talocalcaneal (TC) - Calcaneonavicular (CN)

Triple Arthrodesis is the fusion of ____ - Talonavicular joint - Talocalcaneal joint - Calcaneocuboid joint Fixations used in triple arthrodesis - 6.5 or 7.0 mm cannulated screw for the STJ - 2 staples at 90 for the CC and TN joints - Valgus

Hindfoot should be positioned in slight _____ when perform triple arthrodesis Order of resection and fixation in triple arthrodesis

- Resect MTJ (CC then TN) allows access to the STJ - Resect STJ - Temporarily fixate STJ - Temporarily fixate MTJ (TN then CC) - Check with C-arm - Fixate STJ - Fixate MTJ - Limitation of joint movement - Laterally in the sinus tarsi just distal to the posterior facet of the calcaneus - Limit pronation and reduce heel valgus

Arthroreisis Implant devices are inserted _______ in STJ arthroreisis The goal of STJ arthroreisis implants

114

H. Huynh NBPME Part II Rapid Review Anterior-medial portal - Medial to anterior tibial tendon and lateral to the medial malleolus - Avoid the saphenous nerve and saphenous vein - Lateral to the peroneus tertius tendon and medial to the lateral malleolus - Avoid the superficial peroneal nerve - Lateral to the EHL tendon and medial to the EDL tendon - Avoid anterior tibial artery, deep peroneal nerve - Medial to the Achilles, lateral to the tarsal canal - Avoid the posterior tibial artery, and the tibial nerve - Lateral to Achilles, medial to the fibular malleolus - Avoid the sural nerve, lesser saphenous vein, and the peroneal tendons - Attaches the medial cuneiform to the second metatarsal - Aka- the medial interosseous tarsometatarsals ligament - Strongest interosseous tarsometatarsals ligament - This ligament plus the recessed 2nd metatarsal are responsible for most of the stability at the Lisfrancs joint - Responsible for the avulsion type fracture of the base of the medial aspect of the 2nd metatarsal

Anterior-lateral portal

Anterior-central portal

Posterior-medial portal

Posterior-lateral portal

Lisfrancs ligament

115

H. Huynh NBPME Part II Rapid Review Sequential reduction for hammertoe - Long extensor hood recession - Long extensor tenotomy - PIPJ capsulotomy and arthroplasty or arthrodesis - MPJ capsulotomy - MPJ flexor plate release - Z-plasty or V-Y skin plasty - Z-tendon lengthening - Release extensor hood - Capsulotomy (dorsally and medially) - Plantar plate release - Plantar skin wedge excision - Incomplete fracture in which cortex on only one side is disrupted; seen in children due to their soft bones - Calcaneofibular ligament is most likely damaged - Rupture of this ligament may also tear the peroneal tendon sheath - Account for 95% of ankle sprains - Talofibular ligament is most likely damaged - Anterior talofibular ligament - Anterior displacement = 2 cm as compared to the contralateral side indicates a ruptured - Posterior talofibular ligament - Resists posterior displacement of the talus in the ankle mortise - Rarely ruptures, only in severe traumatic accident

Sequential release for an overlapping 5th toe

Greenstick fracture

Inversion ankle sprain with foot dorsiflexed

Inversion ankle sprain with foot plantarflexed

Anterior draw radiograph

116

H. Huynh NBPME Part II Rapid Review Stress inversion radiograph - Calcaneofibular ligament - Talar tilt of > 5 degree to the contralateral side indicates a ruptured - Watson-Jones - Lee - Evans - Christmas and Snook - Elmslie - Peroneal subluxation/ dislocation classification - Grade I Most common, the retinaculum separates from the fibrocartilagenous ridge - Grade II Involves the fibrocartilagenous ridge along with the retinaculum detaching from the fibula - Grade III Least common, involves an avulsion fracture of fibula - Malignant hyperthermia

One ligament repair procedures

Two ligaments repair procedures

Eckert and Davis Classification

This condition most frequently seen when receiving a combination of muscle relaxant (succinylcholine) and inhalation general anesthetic (halothane) Rule of nines

- Used to estimate the percentage of body burned in adults - Anterior Head: 4.5 % L arm: 4.5%; R arm: 4.5% Chest: 18% Genitalia: 1% L leg: 9%; R Leg: 9% - Posterior Head: 4.5% L arm: 4.5%; R arm: 4.5%

117

H. Huynh NBPME Part II Rapid Review Back: 18% L leg: 9%; R leg: 9% Johnson and Strom Classification - Tibialis posterior tendon dysfunction classification - Stage 1 Tendon length normal - Stage 2 Tendon elongated, hindfoot mobile - Stage 3 Tendon elongated, hindfoot deformed and stiff

Changes associated with stage 1 PT tendon dysfunction - Tendon condition - Hindfoot - Pain - Single-heel-rise test - Too-many-toes sign with forefoot abduction - Pathology - Treatment

- Peritendinitis and/or tendon degeneration - Mobile, normal alignment - Medial: focal, mild to moderate - Mild weakness - Normal - Synovial proliferation, degeneration - Conservative, 3 months; surgical, 3 months with synoectomy, tendon debridement, rest

Changes associated with stage 2 PT tendon dysfunction - Tendon condition - Hindfoot - Pain - Single-heel-rise test - Too-many-toes sign with forefoot abduction - Pathology - Treatment

- Elongation - Mobile, valgus position - Medial: along PTT, moderate - Marked weakness - Positive - Marked degeneration - Transfer FDL for PTT

118

H. Huynh NBPME Part II Rapid Review Changes associated with stage 3 PT tendon dysfunction - Tendon condition - Hindfoot - Pain - Single-heel-rise test - Too-many-toes sign with forefoot abduction - Pathology - Treatment Normal axis of EKG

- Elongation - Fixed, valgus position - Medial: possibly lateral, moderate - Marked weakness - Positive - Marked degeneration - Subtalar arthrodesis - Upright QRS in leads I and leads aVF - Double thumbs-up sign - Right bundle branch block (RBBB)

Bradyarrhythmia in COPD patients is the result of ___________________ Etiology of atrial fibrillation

- PIRATES Pulmonary disease Ischemia Rheumatic heart disease Anemia/ Atrial myxoma Thyrotoxicosis Ethanol Sepsis - Peak T waves, widening of PR interval, widening of QRS, and sinusoidal wave pattern ventricular fibrillation - Sinus tachycardia - S1Q3T3 (S wave in lead I, Q wave in lead II, and an inverted T wave in lead III) - WiLLiaM MaR-RoW - W pattern of QRS in V1-V2 and M pattern of QRS in V3-V6 LBBB

Hyperkalemia in EKG

Pulmonary embolus in EKG

To distinguish between left bundle branch block and right bundle branch block in EKG

119

H. Huynh NBPME Part II Rapid Review - M pattern of QRS in V1-V2 and W pattern of QRS in V3-V6 for RBBB Management of atrial fibrillation - ABCD - Anticoagulate - Beta blockers to control rate - Cardiovert/ Calcium channel blockers - Digoxin - Dilated - Hypertrophic - Restrictive - ABCDs - Alcohol - Beriberi (wet) - Coxsackie B virus - Chagas disease - Cocaine - Doxorubicin - S3 end of rapid ventricular filling

Three types of cardiomyopathy

Known causes of dilated cardiomyopathy

Heart sound associated with dilated cardiomyopathy The most common cause of sudden death in young, healthy athletes

- Hypertrophic cardiomyopathy (myocardium that is slow to relax)

Heart sound associated with hypertrophic cardiomyopathy

- S4 high atrial pressure/ stiff ventricle; atrial kick associated with a hypertrophic ventricle - Myocardium that has impaired elasticity - Caused by infiltrative disease or by scarring and fibrosis - Congestive heart failure (CHF)

Restrictive cardiomyopathy

Clinical manifestation of cardiomyopathy

120

H. Huynh NBPME Part II Rapid Review Risk factors of CHF - CAD - Hypertension - Valvular heart disease - Pericardial disease - Cardiomyopathy - Pulmonary hypertension - FAILURE - Forgot medication - Arrhythmia/ Anemia - Ischemia/ Infarct/ Infection - Lifestyle (most common cause; e.g. increase sodium intake, decrease exercise) - Upregulation (increase cardiac output; e.g. pregnancy, hyperthyroidism) - Renal failure (fluid overload) - Embolus (pulmonary) - ABCD - Asthma - Block (heart block) - COPD - Diabetes - CAPTOPRIL - Cough - Angioedema - Potassium excess - Taste changes - Orthostatic hypotension - Pregnancy is contraindicated - Rash - Indomethacin inhibition - Liver toxicity

Causes of recurrent CHF

Caution when using Beta blockers to treat CHF

Side effects of ACEIs

121

H. Huynh NBPME Part II Rapid Review Right-sided CHF symptoms - JVD - Hepatomegaly - Hepatojugular reflux - Bipedal edema - Bilateral basilar rales - S3 gallop - Pleural effusions - Pulmonary edema - Left-sided heart failure

Left-sided CHF symptoms

The most common cause of right-sided heart failure is _______________ Acute CHF management

- LMNOP - Lasix - Morphine - Nitrates - Oxygen - Pulmonary ventilation - Clinical manifestations include: - Stable and unstable angina - Shortness of breath - Dyspnea on exertion - Arrhythmias - MI - Heart failure - Sudden death - Age - Gender (males > females) - Hypercholesterolemia - DM - Hypertension - Obesity - Smoking - Positive family history

Coronary artery disease (CAD)

Major risk factors for CAD

122

H. Huynh NBPME Part II Rapid Review The statins lipid lowering agents belong to which class? Side effects/ contraindications of HMG-CoA reductase inhibitors - HMG-CoA reductase inhibitors

- HMG-CoA - Hepatotoxicity - Myositis - Girl in pregnancy/ Growing children - Coumadin/ Cyclosporine interactions - Substernal chest pain - Worsened by exertion - Relieved by rest or nitrates - Troponin appear first, most sensitive and specific - CK with CK-MB fraction appear after troponin - DUST - Depressed ventricular function - Unable to perform PTCA (percutaneous transluminal coronary angioplasty) - Stenosis of left main - Triple-vessel disease - Systolic BP > 140 and/or diastolic BP > 90 - Primary hypertension is the most common cause of high blood pressure - Treatment: ABCD ACEIs/ ARBs Beta blockers Calcium channel blockers Diuretics

The classic triad of angina

Serial cardiac enzymes for myocardial infarction (MI)

Indications to perform CABG (coronary artery bypass graft) on MI patient

Hypertension

123

H. Huynh NBPME Part II Rapid Review Side effects of thiazides - Hyper-GLUC - HyperGlycemia - HyperLipidemia - HyperUricemia - HyperCalcemia - CHAPS - Cushings syndrome - Hyperaldosteronism - Aortic coarctation - Pheochromocytoma - Stenosis of renal arteries - Inflammation of pericardial sac, often with effusion - Look for signs of PERICarditis - Pulsus paradoxus - EKG changes - Rub - Increase JVP - Chest pain - CARDIAC RIND - Collagen vascular disease - Aortic dissection - Radiation - Drugs - Infections - Acute renal failure (uremia) - Cardiac (MI) - Rheumatic fever - Injury - Neoplasms - Dresslers syndrome - Excess fluid in the pericardial sac compromised ventricular filling and decrease cardiac output

Causes of secondary hypertension

Percarditis

Causes of pericarditis

Cardiac temponade

124

H. Huynh NBPME Part II Rapid Review - More closely related to the rate of fluid formation - Presents with fatigue, dyspnea, tachycardia, and tachypnea that can rapidly cause shock and death - Examination may reveal Becks triad, narrow pulse pressure, pulsus paradoxus, and Kussmauls sign Becks triad for cardiac temponade - 3 Ds - Distant heart sounds - Distended jugular veins (distended neck veins) - Decreased arterial pressure (hypotension) - Jugular venous distention on inspiration - Exertional Dyspnea - Angina - Syncope - SAD BET - Sex (male) - Age - Diabetes - BP high - Elevated cholesterol - Tobacco - Atherosclerosis

Kussmauls sign

Classic triad of aortic stenosis

Risk factors for atherosclerosis

Aortic aneurysm is most commonly associated with ____________ Aortic dissection is most commonly associated with ______________

- Hypertension

125

H. Huynh NBPME Part II Rapid Review Trousseaus syndrome - Hypercoagulability due to malignancy (usually adenocarcinoma) - Occlusion of the blood supply to the extremities by atherosclerotic plaques - Initially presents with intermittent claudication - Most often caused by embolization from the heart - 6 Ps - Pain - Pallor - Pulselessness - Paralysis - Paresthesia - Poikilothermia (cold) - Sawtooth P waves - Angina is new, is worsening, or occurs at rest - ACEI

Peripheral vascular disease (PVD)

Acute ischemia

Classic EKG finding in atrial flutter Unstable angina

Antihypertensive for a diabetic patient with proteinuria Treatment for atrial fibrillation

- Anticoagulation, rate control, cardioversion - Immediate cardioversion - Dresslers syndrome: fever, pericarditis, increase ESR - Echocardiogram

Treatment for ventricular fibrillation Autoimmune complication occurring 2-4 weeks post MI Diagnostic test for hypertrophic cardiomyopathy

126

H. Huynh NBPME Part II Rapid Review A fall in systolic BP of > 10mmHg with inspiration Classic EKG findings in pericarditis - Pulsus paradoxus (seen in cardiac temponade) - Low-voltage, diffuse ST-segment elevation - ST segment elevation (depression mean ischemia), flattened T waves, and Q waves - Prinzmetals angina

EKG findings suggesting MI

A young patient has angina at rest with ST-segment elevation. Cardiac enzyme are normal

Common symptoms associated with silent - CHF, shock, and altered mental status MI The diagnostic test for pulmonary embolism An agent that reverses the effects of heparin The coagulation parameter affected by Warfarin A young patient with a family history of sudden death collapses and dies while exercising Atopic dermatitis/ eczema - V/Q scan

- Protamine

- PT

- Hypertrophic cardiomyopathy

- A relapsing inflammatory skin disorder characterized by pruritus - Persistent scratching lichenification - Eczema is an itch that rashes - ACID - Anaphylactic type I - Cytotoxic type II

Classification of hypersensitivity reactions

127

H. Huynh NBPME Part II Rapid Review - Immune complex type III - Delayed hypersensitivity type IV The allergic triad - Atopic dermatitis - Asthma - Hay fever - Chronic blistering eruptions on an inflamed base - Only on the skin; usually the arms and thighs - Autoantibodies to BP1 and BP2 - Location of Autoantibodies: epidermaldermal junction - Location of blister: subepidermal, deep - Negative Nikolskys sign - Itchy symptoms - An immune-mediated cutaneous disorder that is due to drugs, infection, vaccinations, or malignancy - Exists in a continuum with StevensJohnson syndrome and toxic epidermal necrolysis - Skin biopsy shows perivascular lymphocytes (mostly T cells) and necrotic keratinocytes - Inflammation of the subcutaneous fat that produces tender erythematous nodules, usually on the anterior tibial areas - Lesions result from hypersensitivity reactions to drugs or from infections, Sarcoid, rheumatic fever, or IBD

Bullous pemphigoid

Erythema multiform

Erythema nodosum

128

H. Huynh NBPME Part II Rapid Review This class of drugs are both a precipitating factor and a treatment for erythema nodosum Lichen planus - NSAIDs

- An inflammatory dermatosis involving skin and mucous membranes - Often induced by drugs and strongly associated with HCV - The 4 Ps - Polygonal - Purple - Pruritic - Papules - A T-cell mediated inflammatory disorder epidermal hyperproliferation - A chronic superficial inflammatory disorder thought to be a reaction to Pityrosporum yeast - Suspect HIV in a young person with severe seborrheic dermatitis - A life-threatening exfoliative mucocutaneous disease often caused by a drug-induced immunologic reaction - SJS involves < 10% of body surface area of epidermal separation - TEN involves > 30% of body surface area of epidermal separation - Involvement of 10-30% is overlap SJS-TEN - The leading cause of mortality is sepsis from superimposed bacterial skin infections (s. aureus in early stages; and pseudomonas in later stages)

Psoriasis

Seborrheic dermatitis

Stevens-Johnson Syndrome/ Toxic Epidermal Necrolysis (SJS/TEN)

129

H. Huynh NBPME Part II Rapid Review Vitiligo - Areas of acquired cutaneous depigmentation due to loss of melanocytes. - Has a significant association with autoimmune diseases such as pernicious anemias, thyroid disease, Addisons disease, and type I DM - A rapidly developing infection of skin and fascia that has high mortality without emergent treatment - Caused by group A streptococci, mixed aerobic-anaerobic bacteria, or Clostridium perfringens - Fourniers gangrene necrotizing fasciitis of the perineal region - Asymmetric shape - Borders irregular - Color variegated - Diameter > 6mm - A cutaneous T-cell lymphoma of unknown etiology - Seborrheic keratosis - Psoriasis

Necrotizing fasciitis

ABCDs of melanoma

Mycosis fungoides

Stuck-on appearance skin condition Red plaques with silvery-white scales and sharp margins

The most common type of skin cancer; - Basal cell carcinoma the lesion is a pearly-colored papule with a translucent surface and telangiectasias Honey-crusted lesions - Impetigo

130

H. Huynh NBPME Part II Rapid Review A febrile patient with a history of diabetes presents with a red, swollen, painful lower extremity Positive Nikolskys sign Negative Nikolskys sign A 55 y/o obese patients with dirty, velvety patches on the back of the neck Dermatomal distribution Flat-topped papules Iris-like target lesions A lesion characteristically occurring in a linear pattern in areas where skin comes into contact with clothing or jewelry - Cellulitis

- Pemphigus vulgaris - Bullous pemphigoid - Acanthosis nigricans. Check fasting blood sugar to rule out diabetes - Varicella zoster - Lichen planus - Erythema multiform - Contact dermatitis

Presents with a herald patch, Christmas- - Pityriasis rosea tree pattern A 16 y/o presents with an annular patch of alopecia with broken off, stubby hairs Pinkish, scaling, flat lesions on the chest and back. KOH prep has a spaghettiand-meatballs appearance Premalignant lesions from sun exposure that can cause squamous cell carcinoma Dewdrop on a rose petal - Alopecia areata (autoimmune process)

- Pityriasis versicolor

- Actinic keratosis

- Lesions of primary varicella

131

H. Huynh NBPME Part II Rapid Review Cradle cap - Seborrheic dermatitis. Treat with antifungals

Associated with Propionibacterium acnes - Acne vulgaris and changes in androgens levels A painful, recurrent vesicular eruption of mucocutaneous surfaces Inflammation and epithelial thinning of the anogenital area, predominantly in postmenopausal women Exophytic nodules on the skin with varying degrees of scaling or ulceration; the second most common type of skin cancer Symptoms of diabetic ketoacidosis - Herpes simplex

- Lichen sclerosus

- Squamous cell carcinoma

- DKA - Dehydrated - Ketones/ Kussmaul breathing/ K drops - Acidosis/ Acetone breath - Early-morning hyperglycemia caused by decrease effectiveness of insulin at that time. - Move p.m. insulin closer to bedtime to treat - The single best test for assessing thyroid function - High TSH levels primary hypothyroidism - Low TSH levels primary hyperthyroidism

Dawn phenomenon

TSH measurement

132

H. Huynh NBPME Part II Rapid Review Hyperthyroidism - Causes of thyrotoxicosis: Graves disease, toxic multinodular goiter, and toxic adenomas - Symptoms: weight loss, heat intolerance, nervousness, palpitation, increase bowel frequency, sinus tachycardia or atrial fibrillation, fine tremor, lid lag. - Exophthalmos and Pretibial myxedema are seen only in Graves disease - TSH receptor antibodies are seen in patients with Graves disease - Hashimotos thyroiditis is the most common cause - Symptoms: cold intolerance, constipation, depression, hoarseness, dry/cold/puffy skin, bradycardia - The most popular is Papillary - Papillae (branching) - Palpable lymph nodes -Pupil nuclei (Orphan Annie nuclei) - Psammoma bodies within lesion (often) - Also has a Positive Prognosis - Associated with an autosomal-dominant inheritance - MEN type I (Werners syndrome) - Pancreatic (Zollinger-Ellison syndrome, insulinomas) - Parathyroid - Pituitary tumors - MEN type II (Sipples syndrome) - Medullary carcinoma of the thyroid - Pheochromocytoma - Parathyroid gland hyperplasia

Hypothyroidism

Thyroid neoplasms

Multiple Endocrine Neoplasia (MEN)

133

H. Huynh NBPME Part II Rapid Review - MEN type III (formerly MEN IIB) - Medullary carcinoma of the thyroid - Pheochromocytoma - Oral and intestinal ganglioneuromatosis (mucosal neuromas) - Marfanoid habitus - Pancreas, Pituitary, Parathyroid - Osteoporosis

- MEN type I-III P organs The most common cause of pathologic fractures in elderly, thin women Von Hippel-Lindau syndrome, neurofibromatosis, or MEN II/III syndromes all associated with _______ Pheochromocytoma rule of 10s

- Pheochromocytoma

- 10% extra-adrenal - 10% bilateral - 10% malignant - 10% occur in children - 10% familial - Pressure (hypertension) - Pain (headache) - Perspiration - Palpitations - Pallor/ diaphoresis - Hashimotos thyroiditis

The 5 Ps of pheochromacytoma

The most common cause of hypothyroidism Lab findings in Hashimotos thyroiditis

- High TSH, low T4, antimicrosomal antibodies - Graves disease

Exophthalmos, Pretibial myxedema, and decrease TSH The most common cause of Cushings syndrome

- Iatrogenic steroid administration

134

H. Huynh NBPME Part II Rapid Review The second most common cause of Cushings syndrome A patient presents with signs of hypocalcemia, high phosphorus, and low PTH Stones, bones, groans, psychiatric overtones A patient complaints of headache, weakness, and polyuria; exam reveals hypertension and tetany. Lab reveals hypernatremia, hypokalemia, and metabolic alkalosis A patient presents with tachycardia, wild swings in BP, headache, diaphoresis, altered mental status, and a sense of panic Should alpha or beta antagonists be used first in treating pheochromocytoma? A patient with a history of lithium use presents with copious amounts of dilute urine A postoperative patient with significant pain presents with hyponatremia and normal volume status An antidiabetic agent associated with lactic acidosis A patient presents with weakness, nausea, vomiting, weight loss, and new skin pigmentation. Labs show - Cushings disease

- Hypoparathyroidism

- Signs and symptoms of hypercalcemia

- Primary Hyperaldosteronism (due to Conns syndrome or bilateral adrenal hyperplasia)

- Pheochromocytoma

- Alpha antagonists

- Nephrogenic diabetes insipidus

- SIADH due to stress

- Metformin - Primary adrenal insufficiency (Addisons disease) - Treat with replacement

135

H. Huynh NBPME Part II Rapid Review hyponatremia and hyperkalemia. Treatment are? Goal hemoglobin A1C for a patient with DM Treatment of DKA glucocorticoids, mineralcorticoids, and IV fluids - < 7.0

- Fluids, insulin, and aggressive replacement of electrolytes (e.g. K+) - They can mask symptoms of hypoglycemia - Any error in the design, implementation, or analysis of a study result in conclusions differing from the truth - Observational bias

Why are beta-blockers contraindicated in diabetics? Bias

Bias introduced into a study when a clinician is aware of the patients treatment type Bias introduce when screening defects a disease earlier and thus lengthens the time from diagnosis to death If you want to know if race affects infant mortality rate but most of the variation in infant mortality is predicted by socioeconomic status, then socioeconomic status is a ___________ Sensitivity SnOUT Sensitive test rule OUT disease

- Lead-time bias

- Confounding variable

- The probability that a diseased patient will have a positive test result - True positive / total number of people with the disease - The probability that a nondiseased person will have a negative test result

Specificity

136

H. Huynh NBPME Part II Rapid Review - True negative / total number of people without the disease - SpIN Specific tests rule IN disease Incidence - The number of new cases in a given atrisk population over a specified period of time - The number of existing cases at a moment in time. - Prevalence reflects incidence and disease duration - Out

Prevalence

Sensitive tests have few false negatives and are used to rule _____ a disease PPD reactivity is used as a screening test because most people with TB (except those who are anergic) will have a positive PPD. Highly sensitive or specific? Chronic diseases such as SLE-higher prevalence or incidence Epidemics such as influenza higher prevalence or incidence Case-control study

- Highly sensitive for TB

- Higher prevalence

- Higher incidence

- An observational study in which cases (with disease) and controls (without disease) are identified - Useful for rare diseases or outcomes - Inaccurate due to recall bias and survivorship bias - Prevalence, incidence, and relative risk cannot be calculated; odds ratio can be

137

H. Huynh NBPME Part II Rapid Review Cohort study - An observational study in which a cohort of exposed and nonexposed individuals are followed to determine if disease develops. - Usually prospective, but can also be retrospective - Allows the effects of rare exposures and multiple outcomes of an exposure to be examined - Relative risk, incidence, prevalence, and odds ratio can be determined - Selection bias and confounding variables may complicate result interpretation - A survey of the population at a single point in time - Used to estimate disease prevalence and to form hypotheses - Causal relationships cannot be established - A prospective study in which subjects are assigned to a treatment or control group - Randomization decrease bias and confounding - May be blinded (the patient does not know to which group he/she is assigned) - May be double blinded (neither the patient nor the researcher knows the group assignment) prevent observational bias - Highest quality study - Very costly and time intensive

Cross-sectional study

Randomized controlled clinical trial (RCCT)

138

H. Huynh NBPME Part II Rapid Review Disease Prevention - Primary prevention - Health-promoting measures to decrease the development (decrease the incidence) of disease - The detection of a disease when it is asymptomatic or mild to prevent its complications - The reduction of morbidity associated with the presence of disease - Conflict of interest

- Secondary prevention

- Tertiary prevention

A doctor refers a patient for an MRI at a facility he/she owns Heparin

- Increase PTT, affects the intrinsic pathway, and decrease fibrinogen levels; safe in pregnancy - Increase PT, affect the extrinsic pathway, and decrease vitamin K; teratogenic - X-linked recessive coagulopathies that are due to decrease factor VIII (hemophilia A) or factor IX (hemophilia B) - Unlike von Willebrands disease (vWD), bleeding time in hemophilia A and B is normal because no platelet function abnormality is present - Diagnosis: prolonged PTT, decrease factor VIII and IX levels - The most common hereditary bleeding disorder (autosomal dominant) - Due to deficient or abnormal vWF, which stablizes factor VIII and enhances platelet aggregation/ attachment to injured vascular

Warfarin

Hemophilias

Von Willebrands disease (vWD)

139

H. Huynh NBPME Part II Rapid Review endothelium - Diagnosis: decrease levels of activated factor VIII; low levels of immunoreactive vWF; prolonged bleeding time - Ristocetin cofactor assay measures the ability of vWF to agglutinate platelets in the presence of ristocetin Causes of thrombocytopenia - PLATELETS - Platelet disorders: TTP, ITP, DIC - Leukemia - Anemia - Trauma - Enlarged spleen - Liver disease - EtOH - Toxins (benzene, heparin, aspirin, chemotherapy) - Sepsis - An autoimmune platelet disorder in which IgG autoantibody binds to platelets results in lysis by splenic macrophages - Presents in pediatric patients with acute onset following viral illness - Petechiae suggest platelet deficiency - Diagnosis: - CBC shows thrombocytopenia and possibly anemia - PT and PTT are normal; bleeding time is prolonged; DIC panel is normal - Positive platelet-associated IgG test - Peripheral blood smear reveals megathrombocytes without schistocytes

Idiopathic Thrombocytopenic Purpura (ITP)

140

H. Huynh NBPME Part II Rapid Review The most common inherited course of hypercoagulability Leukemia - Factor V Leiden mutation

- Malignant proliferations of hemotapoietic cells - Categorization is based on cellular origin and on the level of differentiation of neoplastic cells - Acute leukemia proliferations of minimally differentiated blast cells - Chronic leukemia proliferations of more mature, differentiated cells - Lymphocytic leukemia neoplasms of lymphoid cells (B and T lymphocytes) - Myelogenous leukemia neoplasms of myeloid cells (granulocytes, monocytes, platelets, erythrocytes) - LEUKEMIA - Light skin (pallor) - Energy decrease - Underweight - Kidney failure - Excess heat (fever) - Mottled skin (hemorrhage) - Infections - Anemia - Up to age 15: ALL (Acute Lymphocytic Leukemia) - Age 15-59: AML and CML (Acute Myelogenous Leukemia and Chronic Myelogenous Leukemia) - Age 60 and over: CLL (Chronic Lymphocytic Leukemia)

Signs of leukemia

The most common leukemias by age

141

H. Huynh NBPME Part II Rapid Review Acute Lymphocytic Leukemia (ALL) - Most common in children - Presents with malaise, fever, lethargy, and weight loss - Bone pain, infection, and hemorrhage are also seen - Examination reveals lymphadenopathy and hepatosplenomegaly - Associates with Down syndrome (We will ALL go DOWN together) - Pathognomonic of AML: auer rods eosinophilic, needle like cytoplasmic inclusions that are found in blast cells - Back pain - Anemia - Renal insufficiency - TICS - Thalassemia - Iron deficiency - Chronic disease - Sideroblastic anemia

Acute Myelogenous Leukemia (AML)

Triad of multiple myeloma

Causes of microcytic anemia

Blood cell morphologies and associated diseases - Spherocytes - Blister cell - Burr cell - Heinz body - Schistocyte - Target cell

- G6PD deficiency, membranopathy (immune and nonimmune) - G6PD deficiency - Acute renal failure, uremia - Thalassemia, hemoglobinopathies, and enzymopathies (G6PD) - Artificial heart valves and microangiopathic hemolytic anemias - Splenectomy

142

H. Huynh NBPME Part II Rapid Review Features of iron deficiency anemia - Fe (iron) KAP - Fatigue - Exercise tolerance (decrease) - Koilonychia - Angular cheilitis - Pica, Pallor - FAT RN - Fever - Anemia - Thrombocytopenia - Renal dysfunction - Neurologic abnormalities - A group of disorders in which a coagulopathy is associated with a platelet deficiency and RBC hemolysis - Includes: - Thrombotic thrombocytopenic Purpura (TTP) - Hemolytic uremic syndrome (HUS) - Disseminated Intravascular Coagulopathy (DIC) - Peripheral blood smear shows schistocytes (fragmented RBCs) - In DIC, activation of the clothing cascade leads to increase PT/PTT - In TTP and HUS, platelet aggregation plays a larger role and PT/PTT are often normal - SICKLE - Splenomegaly, Sludging - Infection - Cholelithiasis - Kidney hematuria - Liver congestion, leg ulcer - Eye changes

Pentad of Thrombotic Thrombocytopenic Purpura (TTP)

Microangiopathic hemolytic anemia

Signs of sickle cell disease

143

H. Huynh NBPME Part II Rapid Review Sickle cell disease - An autosomal-recessive disease - A defect in the beta-globin chain - Sickle cell crises are precipitated by infection, dehydration, and hypoxia - Decrease quantity of an alpha or beta globin chain - A defect in heme synthesis - Hereditary spherocytosis

Thalassemia

Porphyria The most common inherited hemolytic anemia Hemolytic uremic syndrome triad

- Anemia - Thrombocytopenia - Acute renal failure - Hodgkins lymphoma - Infection - Cancer - Autoimmune disease - PAINT - Pleural - Alveolar - Interstitial lung disease/ Inflammatory/ Idiopathic - Neuromuscular - Thoracic wall - A systemic rheumatologic disease of unknown etiology characterized by noncaseating granulomas - Features: GRUELING - Granulomas - Rheumatoid arthritis - Uveitis

Reed-sternberg cells The three most common causes of fever of unknown origin

Causes of restrictive lung disease

Systemic sarcoidosis

144

H. Huynh NBPME Part II Rapid Review - Erythema nodosum - Lymphadenopathy - Interstitial fibrosis - Negative TB test - Gammaglobulinemia Causes of obstructive pulmonary disease - ABCT - Asthma - Bronchiectasis - Cystic fibrosis - Tracheal or bronchial obstruction - Squamous cell carcinoma

Non-small cell lung cancer associated with hypercalcemia Lung cancer associated with SIADH Lung cancer highly related to cigarette exposure Nephritic syndrome Nephrotic syndrome

- Small cell lung cancer (SCLC) - Small cell lung cancer (SCLC)

- Hematuria, hypertension, and oliguria - Proteinuria, hypoalbuminemia, hyperlipidemia, hyperlipiduria, edema - Membranous glomerulonephritis

The most common form of nephritic syndrome The most common form of glomerulonephritis Glomerulonephritis with deafness Glomerulonephritis with hemoptysis

- IgA nephropathy (Bergers disease)

- Alports syndrome - Wegeners granulomatosis and Goodpastures syndrome - Glomerulonephritis/ nephritic syndrome

Presence of red cell casts in urine sediment

145

H. Huynh NBPME Part II Rapid Review Eosinophils in urine sediment Waxy casts in urine sediment and Maltese crosses (seen with lipiduria) Hematuria, flank pain, and palpable flank mass Three systemic diseases that cause nephrotic syndrome - Allergic interstitial nephritis - Nephrotic syndrome

- Renal cell carcinoma (RCC)

- DM - SLE - Amyloidosis - Formula to calculate fluids for burn patient the 1st 24 hrs - 4mL x weight in kg x % body surface - Give 50% of fluids over the first eight hours and the remaining 50% over the following 16 hours - Folate deficiency

Parkland formula

Macrocytic, megaloblastic anemia without neurologic symptoms Macrocytic megaloblastic anemia with neurologic symptoms The most common organism in burnrelated infections Method of calculating fluid repletion in burn patients

- B12 deficiency

- Pseudomonas

- Parkland formula

146

Вам также может понравиться